Вы находитесь на странице: 1из 119

‫‪Prometric Exam Collection 2019‬‬ ‫ودالشمال‪01‬‬

‫تهويه‪:‬‬
‫ال أبيح وال أسامح أحد فى نسخ واستخدام ملفاتى فى أغراض شخصية‬
‫وسيكون اللقاء بيهها أمام اهلل وتعترب مجيع ملفاتى جمانا وليست للبيع‬

‫اجلاهل يعتكد نفسى حكيماً‪ ،‬أما احلكيم فوو‬


‫من يعرف بأنى أمحل‪.‬‬
‫بالتوفيل للجميع‬

‫قروب طور ذاتك ‪ ..‬مناقشة امتحانات البرومتريك للتمريض‬ ‫‪M- page 1‬‬
‫‪https://www.facebook.com/groups/1639727976293954‬‬
Prometric Exam Collection 2019 01‫ودالشمال‬

.0
Atrial fibrillation

Treatments for atrial fibrillation include medications to control heart rate


and reduce the risk of stroke, and procedures such as cardioversion to
restore normal heart rhythm.

.2
ST elevation

ST elevation = MI
MI patient have = ST elevation and high CK – MB
o Initial medical therapy during STE MI consists of oxygen
administration, antiplatelet therapy (aspirin, thienopyridines
andglycoprotein IIb/IIIa inhibitors), anticoagulation (heparin or
bivalirudin), anginal pain

‫ مناقشة امتحانات البرومتريك للتمريض‬.. ‫قروب طور ذاتك‬ M- page 2


https://www.facebook.com/groups/1639727976293954
Prometric Exam Collection 2019 01‫ودالشمال‬

.3
A 37 years old pregnant women during the labour shoulder dystocia
occur, the doctor ask the nurse to help him, what the nurse do?
Make a pressure on supra pubic area to women.

.4
A 30 year-old woman with type 1 diabetes mellitus receives mixed type
of insulin in the morning and before bed time. She reports that the level
of her fasting blood sugar is constantly high when she checks it every
morning at home.
Which dose of insulin is most likely causing this problem?
a. Low morning, regular insulin
b. High morning NPH1
c. High evening regular insulin
d. Low evening NPH insulin

‫ مناقشة امتحانات البرومتريك للتمريض‬.. ‫قروب طور ذاتك‬ M- page 3


https://www.facebook.com/groups/1639727976293954
Prometric Exam Collection 2019 01‫ودالشمال‬

.5
Young patient arrived to ER with burn in his left arm and his weight 48
kg, how calculate fluids requirement for the patient according to
parkland formula.

a. 1527 ml
b. 1220 ml
c. 1728 ml
d. 2000 ml

.6
A 7-week-old infant boy is admitted with projectile vomiting decreased
urine output, decreased bowel movements and weight loss. He has poor
turgor and appears hungry. The nurse observes left-to right peristaltic
waves after he vomits. The nurse would expect to find which of the
following during the physical assessment?
a. Hepato-spleenomegaly
b. A palpable pyloric mass
c. Lymphadenopathy
d. Bulging fontanelles

‫ مناقشة امتحانات البرومتريك للتمريض‬.. ‫قروب طور ذاتك‬ M- page 4


https://www.facebook.com/groups/1639727976293954
Prometric Exam Collection 2019 01‫ودالشمال‬

.7
Best time of Repair of cleft lip and palate surgery

Cleft palate repair ‫ـــــ‬within the 6 to 18 months of age

Cleft lip repair — within the first 2 to 6 months of age

.8
Position of cleft lip and palate after surgery
 Cleft lip repair should be positioned on their side or back to keep
them from rubbing their face in the bed
 Cleft palate in prone position
Elbow restrain used to avoid the child putting his finger in the mouth

‫ مناقشة امتحانات البرومتريك للتمريض‬.. ‫قروب طور ذاتك‬ M- page 5


https://www.facebook.com/groups/1639727976293954
Prometric Exam Collection 2019 01‫ودالشمال‬

.9
Fundus height in gestation age in 32 weeks

.01
When developing the plan of care for a client with suicidal ideation
which of the following would the nurse anticipate as the priority ?

a. Self-steam
b. Sleep
c. Hygiene
d. Safety

.00
A nurse in the postpartum unit is caring for a client who has just
delivered a newborn infant following a pregnancy with placenta previa.
The nurse reviews the plan of care and prepares to monitor the client
for which of the following risks associated with placenta previa?

a. Disseminated intravascular coagulation


b. Chronic hypertension
c. Infection
d. Haemorrhage

‫ مناقشة امتحانات البرومتريك للتمريض‬.. ‫قروب طور ذاتك‬ M- page 6


https://www.facebook.com/groups/1639727976293954
Prometric Exam Collection 2019 01‫ودالشمال‬

Because the placenta is implanted in the lower uterine segment, which


doesn’t contain the same intertwining musculature as the fundus of the
uterus this site is more prone to bleeding.
.02
After tonsillectomy, a child begins to vomit bright red blood .the initial
nursing action is to?
a. Notify the physician
b. Turn the child to the side
c. Maintain an NPO status
d. Administer the prescribed antiemetic
.03
Colour of meconium in first day of labour

A few days after birth, babies pass their first bowel movement which is
called meconium. This sticky, tar-like substance is green-black in colour,
and is simply waste produced in the baby's intestine during pregnancy.
Sometimes babies pass meconium before or during labour

.04
Position maintained post vitrectomy

a. Lithotomy

b. Prone

c. Semi-fowler’s

d. Supine

.05
The patient had undergone spinal anesthesia for appendectomy. To
prevent spinal headache, the nurse should place the patient in which of
the following positions?

a. Semi-Fowler’s
b. Flat on bed for 6 to 8 hours
c. Prone position
d. Trendelenburg’s position

‫ مناقشة امتحانات البرومتريك للتمريض‬.. ‫قروب طور ذاتك‬ M- page 7


https://www.facebook.com/groups/1639727976293954
Prometric Exam Collection 2019 01‫ودالشمال‬

.06
Marginal placenta Previa >>>>> Painless bleeding

After 18-20 weeks gestation

Abdomen soft and non-tender

 Marginal Placenta: Placenta within 2 cm of internal os does not cover


 Partial Previa: Placenta covers internal os when closed does not cover
os when fully dilated
 Complete Previa (Central Previa): Placenta covers internal os even
when fully dilated

‫ مناقشة امتحانات البرومتريك للتمريض‬.. ‫قروب طور ذاتك‬ M- page 8


https://www.facebook.com/groups/1639727976293954
Prometric Exam Collection 2019 01‫ودالشمال‬

.07

‫ مناقشة امتحانات البرومتريك للتمريض‬.. ‫قروب طور ذاتك‬ M- page 9


https://www.facebook.com/groups/1639727976293954
Prometric Exam Collection 2019 01‫ودالشمال‬

.08
In the summer months, a five year-old girl presents with a sore throat
and a dry cough that has slowly become worse over the past three
weeks. Her body temperature is 38.0oC. On auscultation, there is
wheezing and shortness of breath. She lives in an overcrowded house
with three brothers, parents and grandparents in a low-income
neighbourhood where she attends school. Which is the greatest risk
factor?
a. Residing in a low-income neighbourhood
b. Exposure to a pathogen in summer season
c. Attending school
d. Living in crowded conditions
.09

Assessment steps

Inspection palpation Auscultation percussion

Assessment if abdomen included:

Inspection – Auscultation – percussion – palpation

If the abdomen is not included in

Inspection – percussion – palpation – auscultation

.21
Child has Sickle cell anemia and his parent avoided him to play with
other child in out of house, what the causes?

 To decrease oxygen consumption


 Dehydration increases the risk of a sickle cell crisis
.20
Characteristic of democratic leadership

 Group members are encouraged to share ideas and opinions


 Even though the leader retains the final say over decisions.
 Members of the group feel more engaged in the process.

‫ مناقشة امتحانات البرومتريك للتمريض‬.. ‫قروب طور ذاتك‬ M- page 10


https://www.facebook.com/groups/1639727976293954
Prometric Exam Collection 2019 01‫ودالشمال‬

 Creativity is encouraged and rewarded


 Good outcome

.22
Complication of thyroidectomy

a. Distension
b. Vocal cord injury
c. Bleeding
Complications

Major postoperative complications include wound infection, bleeding,


airway obstruction (compressing hematoma, tracheomalacia),
hypocalcaemia, thyroid storm (uncommon, usually associated with Grave's
disease) and recurrent laryngeal nerve injury

.23
Side effects of chemotherapy
Common side effects of chemotherapy include fatigue, nausea,
diarrhoea, mouth sores, hair loss, and anemia.
A drop in the levels of any of these blood cell counts results in specific
side effects. Fatigue is one of the most common side effects of
chemotherapy.

.24
What the first activity for new head nurse

a. Assessment activity of group


b. Planning new strategy
c. Change staff
d. Ask plan of previous nurse.

.25
Patient with congestive heart failure and have oedema

Diuretic, weighting, Intake and output chart

‫ مناقشة امتحانات البرومتريك للتمريض‬.. ‫قروب طور ذاتك‬ M- page 11


https://www.facebook.com/groups/1639727976293954
Prometric Exam Collection 2019 01‫ودالشمال‬

.26
Sign and symptom of chickenpox

 An itchy rash is the most common symptom of chickenpox.


 The infection will have to be in the body for around 7 to 21 days
before the rash and other symptoms develop.
 The non-rash symptoms may last a few days and include:
 fever

 headache

 loss of appetite

.27
The immunization for child in 6 months >>>>> Oral polio vaccine

.28
Indication of folic acid

Folic acid is a pregnancy important Taking a prenatal vitamin with the


recommended 400 micrograms (mcg) daily of folic acid before and during
pregnancy can help prevent birth defects of your baby's brain and spinal
cord

.29
The nurse begins a morning shift with the following patients waiting to
see by doctor in the waiting room. Which patient has more priority to
be seen first?
Patients waiting
A A 56 year-old man who has used all his diuretic and is coughing
up sputum
B A 47 year-old woman who was hospitalized three days previously
with pneumonia
C A 22 years female with cough and headache discharge from hospital
two days ago.
D A 37 year-old man who has a history of deep vein thrombosis and
taking daily heparin

‫ مناقشة امتحانات البرومتريك للتمريض‬.. ‫قروب طور ذاتك‬ M- page 12


https://www.facebook.com/groups/1639727976293954
Prometric Exam Collection 2019 01‫ودالشمال‬

.31
Two nurses in the general ward, one of them have experience and other
new nurses, student under training talk with new nurse no different
between them, the nurse should explain to the student:

Nurse registered and student under training

Nurse responsible and have accountability

.30
Patient with fecal impaction, how to detect the disease?
a. fecal with blood
b. rectal examination
c. endoscopy
A fecal impaction is a solid, immobile bulk of feces that can develop in the
rectum as a result of chronic constipation

Diagnostic tests:

 X-ray of the abdomen.


 Abdominal ultrasound
 Sigmoid scope.
 Barium enema
 X-ray of the colon and rectum.

.32
Side effects of lithium treatment = electrolyte imbalance

 Lithium can cause nausea, diarrhoea, dizziness, muscle


weakness, fatigue, and a dazed feeling. These unwanted side effects
often improve with continued use.
 Fine tremor, frequent urination, and thirst can occur and may persist
with continued use.
 Weight gain and swelling from excess fluid can also occur

‫ مناقشة امتحانات البرومتريك للتمريض‬.. ‫قروب طور ذاتك‬ M- page 13


https://www.facebook.com/groups/1639727976293954
Prometric Exam Collection 2019 01‫ودالشمال‬

.33
A patient after the operation with a severe pain, the nurse gives
analgesic as order and after that she assess patient pain, which phase of
nursing process
The 5 Steps of the Nursing Process:
 Assessment Phase – first step of the nursing process is assessment.
During this phase, the nurse gathers information about a patient's
psychological, physiological, sociological, and spiritual status.
 Diagnosing phase – involves a nurse making an educated judgment about
a potential or actual health problem with a patient.
 Planning Phase – Once a patient and nurse agree on the diagnoses, a plan
of action can be developed. If multiple diagnoses need to be addressed
 Implementing phase – is where the nurse follows through on the decided
plan of action. This plan is specific to each patient and focuses on
achievable outcomes.
 Evaluation Phase – Once all nursing intervention actions have taken
place, the nurse completes an evaluation to determine of the goals for
patient wellness have been met
.34
 Common side effects of Clomid include:
 abnormal vaginal/uterine bleeding,
 breast tenderness or discomfort,
 headache,
 nausea, vomiting,
 Diarrhea,
 flushing,
 Blurred vision or other visual disturbances

‫ مناقشة امتحانات البرومتريك للتمريض‬.. ‫قروب طور ذاتك‬ M- page 14


https://www.facebook.com/groups/1639727976293954
Prometric Exam Collection 2019 01‫ودالشمال‬

.35
Diet of Nephrotic syndrome

 A healthy diet for Nephrotic Syndrome patients consists of low salt,


low fat and low cholesterol, with emphasis on fruits and vegetables.
 Fluid and protein restriction
NOTE: The amount of protein and fluid a patient with Nephrotic Syndrome
should have depends on the patient's current condition, age and weight.

.36
Patient post pericardiocentesis and have Cather what purpose of the
catcher?

a. Assess wound
b. drainage
c. healing

 Drains this fluid and prevents future fluid build up


 Uses the catheter to drain excess fluid.
 The catheter may come right out after the procedure

.37
Baby have nose flattened. Ear short expect have.

a. Cushing disease
b. Down syndrome

‫ مناقشة امتحانات البرومتريك للتمريض‬.. ‫قروب طور ذاتك‬ M- page 15


https://www.facebook.com/groups/1639727976293954
Prometric Exam Collection 2019 01‫ودالشمال‬

.38
Which of the following is described as premature separation of a
normally implanted placenta during the second half of pregnancy
usually with severe haemorrhage?
a. Placenta Previa
b. Ectopic pregnancy
c. Incompetent cervix
d. Abruptio placentae
.39
A patient reports the onset of a vaginal discharge that is copious, foamy,
foul smelling, and Yellowish green colour. Which organism is most
likely causing the symptoms?
a. Candida alibicans
b. Human papillomavirus
c. Treponeme pallidum
d. Trichomonas
.41
A co-worker informs that the nurse about experiencing increased level
of stress associated with daily responsibilities to help cope with
professional stress, the nurse should encourage the co-worker to;
a. Make a list of unfinished tasks
b. complete complex mental task before physical tasks
c. Acknowledge daily accomplishments
d. Spend time with colleague away from work
.40
Complication of COPD = Pneumothorax

Complications of Chronic Obstructive Pulmonary Disease


 pulmonary hypertension and right-sided heart failure
o decreased amounts of oxygen in the blood causing blood
vessels to constrict
o Lower extremity edema (swelling)
 Acute Exacerbations
Acute exacerbations of COPD are characterized by an abrupt increase of
symptoms.

‫ مناقشة امتحانات البرومتريك للتمريض‬.. ‫قروب طور ذاتك‬ M- page 16


https://www.facebook.com/groups/1639727976293954
Prometric Exam Collection 2019 01‫ودالشمال‬

o Cough and sputum production increases.


o Wheezing is often increased or present
o Dyspnea (shortness of breath) is increased
 End-stage Lung Disease
o Decline in lung function and rising levels of carbon dioxide in
the blood.
o The increasing carbon dioxide creates a narcotic effect in the
patient, who slowly loses consciousness and stops breathing.
 Pneumonia
o Caused by bacterial infection can lead to respiratory failure in
these patients.
o Streptococcus pneumonia is the most common cause of
bacterial pneumonia in patients with COPD.
 Pneumothorax
 Polycythaemia
o Decreased amounts of blood oxygen by increasing the
production of oxygen-carrying red blood cells.

.42
The rectal temperature, a core temperature, is considered to be one of
the most accurate routes. How many centimetres should the nurse
inserts the thermometer through the rectal?
a. 1, 5 cm
b. 2 cm
c. 2.5 cm
d. 1 cm
Insert it 1/2 inch to 1 inch into the anal opening = 2.5 cm

.43
Post-operative position for pediatric of cleft lip and palate

 A child who has had a cleft lip repair should be positioned on their
side or back to keep them from rubbing their face in the bed.
 A child with only a cleft palate repair may sleep on their stomach
(prone position).

‫ مناقشة امتحانات البرومتريك للتمريض‬.. ‫قروب طور ذاتك‬ M- page 17


https://www.facebook.com/groups/1639727976293954
Prometric Exam Collection 2019 01‫ودالشمال‬

.44
Method to prevent drug back = z-track technique

Z-tracking is done almost the same way a person would give a normal IM
injection

Z-track method used to prevent the back-tracking of medications and


leakage of the medications into the surrounding tissue

.45
Patient has severe pain given analgesic but have resistance pain, what
action for patient? Give morphine

Severe pain gives narcotic as order


.46
Mode of transmission

MRSA = contact precaution


Smallpox = Airborne isolation
TB = Airborne isolation = N95 mask
Staphylococcus = droplet
Ebola virus = Contact

.47
cyanotic spell for child = chest knee position

 The 'Tet spell' (also called 'hypoxic spell', 'cyanotic spell',


'hypercyanotic spell', 'paroxysmal dyspnea') is an episodic
central cyanosis due to total occlusion of right ventricle outflow in a
patient with a congenital heart disease

‫ مناقشة امتحانات البرومتريك للتمريض‬.. ‫قروب طور ذاتك‬ M- page 18


https://www.facebook.com/groups/1639727976293954
Prometric Exam Collection 2019 01‫ودالشمال‬

 This is called cyanosis. This is due to a low level of oxygenated blood


flowing around the body. The level of the child's cyanosis depends on
the narrowness of the pulmonary valve.
 Knee chest position- To increase systemic vascular resistance
 Oxygen- to improve oxygenation. It is better given in nebulized form
.
 Morphine- Depresses respiratory center→ Decreases Systemic
Venous Return. While giving this, facilities for ventilation should be
available.
.48
Nursing role after epidural anaesthesia

Epidural analgesia is the administration of opioids and/or local anesthetics


into the epidural space. ... While epidural analgesia is used to manage
pain, epidural anesthesia is used to provide anaesthesia during labour and
delivery and surgical procedures

 Assess and monitor patients carefully and report


 Monitor respiratory status

.49
Complication of postpartum Haemorrhage
 Thromboembolic disorders
 Sub involution of uterus
 Infections
 Depression
 Shock
 Death
.51
28 year-old married woman is admitted in the Genecology Ward for the
observation after her miscarriage. She has not been able to sustain any
pregnancy since she got married for the past three years and she had
three miscarriages during this time. What intervention is the most
appropriate?
a. Family history
b. Past medical history

‫ مناقشة امتحانات البرومتريك للتمريض‬.. ‫قروب طور ذاتك‬ M- page 19


https://www.facebook.com/groups/1639727976293954
Prometric Exam Collection 2019 01‫ودالشمال‬

c. Physical examination
d. Laboratory investigations

.50
Patient comes to ER with hyperthermia and doctor order to gives
aspirin 50 milligrams, after 2 hours the checked temperature and found
high, that due to non-absorption of aspirin, what cause of this problem?

a. Short times of duration

b. Inadequate dose

c. high acid in stomach

 Used to reduce pain and swelling in conditions such as arthritis.


 Non-steroidal anti-inflammatory drug (NSAID).
 Aspirin was not absorbed by the stomach at pH 6.5 high acid
.52
Dexamethasone for preterm = enhance lung maturation

Dexamethasone

 Accelerates maturation of fetal lungs


 decrease number of neonates with respiratory distress syndrome
 Improves survival in preterm delivered neonates.
 Optimal gestational age for use of dexamethasone therapy is 31 to 34
weeks of gestation
.53
When administering albuterol to a child with asthma, the nurse should
observe for sign of what major side effect to this medication?
a. Tachycardia
b. Renal failure
c. Apnea
d. Blurred vision
.54
A process of heat loss which involves the transfer of heat from one
surface to another without contact is:

‫ مناقشة امتحانات البرومتريك للتمريض‬.. ‫قروب طور ذاتك‬ M- page 20


https://www.facebook.com/groups/1639727976293954
Prometric Exam Collection 2019 01‫ودالشمال‬

a. Evaporation
b. Conduction
c. Convection
d. Radiation
.55
Heat loss mechanism in new born
 Radiation
 Conduction
 Convention
 Evaporation

‫ مناقشة امتحانات البرومتريك للتمريض‬.. ‫قروب طور ذاتك‬ M- page 21


https://www.facebook.com/groups/1639727976293954
Prometric Exam Collection 2019 01‫ودالشمال‬

.56
Sign and symptom of intussusception disease >>>> red jelly stool
Intussusception is a medical condition in which a part of the intestine folds
into the section next to it. It typically involves the small bowel and less
commonly the large bowel
Symptoms:
Abdominal pain:
 With a sudden, loud cry. They may also pull their knees up to their
chests.
 The pain happens at first, usually every 15 to 20 minutes
 Symptoms differ from child to child. Some have no pain at all.
Signs:
 A visible lump or swelling in the stomach
 ―Currant jelly" stool, or stool mixed with blood and mucus red jelly
stool
 Diarrhea
 Fever
 Lack of energy
 Vomiting up bile, a yellow-green fluid

‫ مناقشة امتحانات البرومتريك للتمريض‬.. ‫قروب طور ذاتك‬ M- page 22


https://www.facebook.com/groups/1639727976293954
Prometric Exam Collection 2019 01‫ودالشمال‬

.57
A nurse is performing an assessment of a client who is scheduled for a
caesarean section. Which assessment finding would indicate a need to
contact the physician?

a. Blood pressure 128/76


b. White blood cell count of 12.000
c. Oxygen saturation 96
d. Temperature 37.2c

.58
Purpose of Maslow’s hierarchy of needs = priority of care

.59
Fertility tests for men and women
For men

 Semen Analysis
 Chlamydia test

‫ مناقشة امتحانات البرومتريك للتمريض‬.. ‫قروب طور ذاتك‬ M- page 23


https://www.facebook.com/groups/1639727976293954
Prometric Exam Collection 2019 01‫ودالشمال‬

For women:

 Blood tests. A sample of blood can be tested for a hormone called


progesterone to check whether ovulating.
 Chlamydia test. Affect fertility.
 Ultrasounds scan. Used to check ovaries, womb and fallopian
tubes.
 X-ray.
 Laparoscopy.

.61
Contraindications of oral contraceptive pills

Contraindications:

Absolute contraindications
1. Thromboembolic, coronary, cerebrovascular disease.
2. Greater risk for : Women > 35 years of age – Heavy smokers –
Obese
3. Active liver disease
4. Malignancy of genitals/breast
5. Undiagnosed vaginal bleeding
Relative contraindications:
 Hypertension – Diabetes mellitus – Mental illness – Gall bladder
disease
 Drug Interactions:
 Enzyme inducers (Phenytoin) cause contraceptive failure
 Antibiotics (Tetracyclines, ampicillin.) cause contraceptive failure

‫ مناقشة امتحانات البرومتريك للتمريض‬.. ‫قروب طور ذاتك‬ M- page 24


https://www.facebook.com/groups/1639727976293954
Prometric Exam Collection 2019 01‫ودالشمال‬

.60
Signs and symptoms of Down syndrome = flattened nose

‫ مناقشة امتحانات البرومتريك للتمريض‬.. ‫قروب طور ذاتك‬ M- page 25


https://www.facebook.com/groups/1639727976293954
Prometric Exam Collection 2019 01‫ودالشمال‬

.62
Dilatation of cervix 6cm effacement 100% which stage of labour

Labour has three stages:

 The first stage is when the neck of the womb (cervix) opens to 10cm
dilated.
 The second stage is when the baby moves down through the vagina
and is born.
 The third stage is when the placenta is delivered

‫ مناقشة امتحانات البرومتريك للتمريض‬.. ‫قروب طور ذاتك‬ M- page 26


https://www.facebook.com/groups/1639727976293954
Prometric Exam Collection 2019 01‫ودالشمال‬

.63
Among the following statements, which should be given the HIGHEST
PRIORITY?

a. Client is cyanotic
b. Client blood pressure is 60/40
c. Client temperature is 40 degree
d. Centigrade Client is in extreme pain

‫ مناقشة امتحانات البرومتريك للتمريض‬.. ‫قروب طور ذاتك‬ M- page 27


https://www.facebook.com/groups/1639727976293954
Prometric Exam Collection 2019 01‫ودالشمال‬

.64
Priority of nursing care

 Priority setting is an important skill in nursing, and a skill deficit can


have serious consequences for patients.
 Priority setting can be defined as the ordering of nursing problems using
notions of urgency and/or importance, in order to establish a preferential
order for nursing actions

.65
Bleeding during labour
 bleeding during labour may be caused by a low-lying placenta,
placental abruption, rupture of the uterus, or tearing of the cervix
o Closely monitor her vital signs, intake and output, blood loss, pain,
and uterine contractions.
o Administer intravenous (I.V.) fluids and monitor for signs of
shock, such as tachycardia and hypotension
.66

Sudden outbreak of infectious disease un predictably and large number


Epidemic

While the nurse is administering a large volume enema, the patient


complains of cramping. The nurse should:

a. Increase the flow rate


b. Lower the fluid container
c. Elevate the head of the bed
d. Gently massage the abdomen

‫ مناقشة امتحانات البرومتريك للتمريض‬.. ‫قروب طور ذاتك‬ M- page 28


https://www.facebook.com/groups/1639727976293954
Prometric Exam Collection 2019 01‫ودالشمال‬

.67
X-ray shows signs and symptoms of tuberculosis the mode of prevention
air pone prevention

.68
Hepatitis A isolation precautions

‫ مناقشة امتحانات البرومتريك للتمريض‬.. ‫قروب طور ذاتك‬ M- page 29


https://www.facebook.com/groups/1639727976293954
Prometric Exam Collection 2019 01‫ودالشمال‬

.69
Cervical dilatation and fetal heart beat fetal heart beats monitoring

Every 30 minutes during first stage

Every 15 minutes during second stage of labour

.71
Shortage in staffing and problem happened, charge nurse should inform
Nursing supervisor
.70

Definition of assignment

Nursing supervisor give task to nurses according their skills


.72
Subjective data for patient she have uterine fibroids

Subjective data = symptom


The most common Symptoms of uterine fibroids include:
 Heavy menstrual bleeding.

 Menstrual periods lasting more than a week.

 Pelvic pressure or pain.

 Frequent urination.

 Difficulty emptying the bladder.

 Constipation.

 Backache or leg pains

.73
Clomid (Clomiphene) Side Effects and Risks:

Hot Flashes
 Suddenly feel extremely warm

 Experience face flush

‫ مناقشة امتحانات البرومتريك للتمريض‬.. ‫قروب طور ذاتك‬ M- page 30


https://www.facebook.com/groups/1639727976293954
Prometric Exam Collection 2019 01‫ودالشمال‬

 Break out in a sweat

 Have a more rapid heartbeat

Bloating and Abdominal Discomfort


Weight Gain
Mood Swings
 Mood disturbances

 Emotionally sensitive, tearful, depression , anxious

 Infertility

Nausea and Dizziness


Breast Tenderness

.74
Coombs test

 The indirect Coombs test is used in prenatal testing of pregnant


women and in testing prior to a blood transfusion.

 The test detects antibodies against foreign red blood cells. In this
case, serum is extracted from a blood sample taken from the patient

.75
Dissociative disorder

Dissociative disorders (DD) are conditions that involve disruptions or


breakdowns of memory, awareness, identity, or perception.

.76
Way allow chest recoil during minimum cardiac out put

 CPR alone is unlikely to restart the heart. Its main purpose is to restore
partial flow of oxygenated blood to the brain and heart.
 The objective is to delay tissue death and to extend the brief window of
opportunity for a successful resuscitation without permanent brain
damage

‫ مناقشة امتحانات البرومتريك للتمريض‬.. ‫قروب طور ذاتك‬ M- page 31


https://www.facebook.com/groups/1639727976293954
Prometric Exam Collection 2019 01‫ودالشمال‬

.77
Patient suffer from trauma this method to maintain airway clearance
avoided? Head till chin left

 Head-tilt/chin-lift is the preferred method, it can be dangerous to use


on a patient who may have a cervical spine injury.
 Jaw-thrust manoeuvre comes in: it allows you to clear the tongue
from the airway with minimal neck movement, allowing rescue
breaths to be administered

.78
Sign and symptom of basal skull fracture? Racoon eyes

 Bruising behind the ears


 Bruising around the eyes. racoon eyes
 Blood behind the ear drum.
 A cerebrospinal fluid (CSF) leak occurs in about 20% of cases and
can result in fluid leaking from the nose or ear

‫ مناقشة امتحانات البرومتريك للتمريض‬.. ‫قروب طور ذاتك‬ M- page 32


https://www.facebook.com/groups/1639727976293954
Prometric Exam Collection 2019 01‫ودالشمال‬

.79
Patient with projectile vomiting = pyloric stenosis

 Pyloric stenosis is an uncommon condition in infants that blocks food


from entering the small intestine
 Olive-shaped mass in the baby's abdomen

Symptoms

Signs of pyloric stenosis usually appear within three to five weeks after
birth. Pyloric stenosis is rare in babies older than 3 months.

Signs include:

 Vomiting after feeding. The baby may vomit forcefully, ejecting


breast milk or formula up to several feet away (projectile vomiting).
Vomiting might be mild at first and gradually become more severe as

‫ مناقشة امتحانات البرومتريك للتمريض‬.. ‫قروب طور ذاتك‬ M- page 33


https://www.facebook.com/groups/1639727976293954
Prometric Exam Collection 2019 01‫ودالشمال‬

the pylorus opening narrows. The vomit may sometimes contain


blood.

 Persistent hunger. Babies who have pyloric stenosis often want to


eat soon after vomiting.

 Stomach contractions. This is caused by stomach muscles trying to


force food through the narrowed pylorus.

 Dehydration

 Changes in bowel movements. Since pyloric stenosis prevents food


from reaching the intestines, babies with this condition might be
constipated.

 Weight problems. Pyloric stenosis can keep a baby from gaining


weight, and sometimes can cause weight loss.

A patient finds their 2-weeks-old infant unresponsive. The infant is


limp, cyanotic and pale. There is no respiration, while the skin is cold to
the touch. The parent begins resuscitation, and the infant is transferred
to the hospital where the infant expires. The MOST likely diagnosis is:

a. Sudden infant death syndrome


b. Apparent life-threatening event
c. Apnea of infancy
d. Apnea of unknown origin

‫ مناقشة امتحانات البرومتريك للتمريض‬.. ‫قروب طور ذاتك‬ M- page 34


https://www.facebook.com/groups/1639727976293954
Prometric Exam Collection 2019 01‫ودالشمال‬

.81

Position liver biopsy


Patient lies on back and position his right hand above the head on the
table

.80
Advantage of documentation:

a. Cost for hospitals


b. Spent time with patient
c. Decrease of satisfaction
Advantages and Disadvantages of Documents

Advantages Disadvantages
 Access to difficult subjects  May be seen as subjective

 No Hawthorne effect  Can be time consuming

 Low cost and detailed  Cost for hospitals

 Documenting time spent with  Decrease of satisfaction


patient
 Ethical - may not have been made
 Researcher may impose their own for research purpose so need
meanings when interpretating data informed cones

‫ مناقشة امتحانات البرومتريك للتمريض‬.. ‫قروب طور ذاتك‬ M- page 35


https://www.facebook.com/groups/1639727976293954
Prometric Exam Collection 2019 01‫ودالشمال‬

.82
What is affected organs in pyloric stenosis?

a. Stomach and oesophagus


b. Liver and spleen
c. Liver and bile duct
d. Stomach and small intestine

Pyloric stenosis is the thickening of the pylorus muscle present at the


intersection of the stomach and the small intestine.

The pylorus or pyloric part, connects the stomach to the duodenum

.83
Mother missed one oral Contraceptive pill

 Missed the first pill – normal to 5th day of the cycle at the same time.
 During the next 7-10 days, it is better to use additional means of
contraception (barrier or chemical) since pregnancy is possible in this
situation.
 first two weeks of taking the pills, need to quickly take the missed
birth control pill and then use additional means of contraception for
the next few days to protect yourself

‫ مناقشة امتحانات البرومتريك للتمريض‬.. ‫قروب طور ذاتك‬ M- page 36


https://www.facebook.com/groups/1639727976293954
Prometric Exam Collection 2019 01‫ودالشمال‬

.84
New nurse come to hospital?

a. Review with her job description


b. Ask her about number of procedure that he did before
.85
New head nurse what is first action?

a. Meeting with staff


b. Planning
c. Start to change immediately
.86
The nurse gives mother health education about breast feeding. The mother
asks the nurse why the breast feeding is better than other milk.
a. Economic and contain immunity
b. Compared with cow's milk, the mother's milk is faster to absorb
c. The child has a minimal amount of breast milk compared to other types
.87
Complication of encephalohematoma

 Tearing of nearby blood vessels


 Nerve damage
 Infection
 Haemorrhage / bleeding
A cephalhaematoma is a haemorrhage of blood between the skull and the
periosteum

‫ مناقشة امتحانات البرومتريك للتمريض‬.. ‫قروب طور ذاتك‬ M- page 37


https://www.facebook.com/groups/1639727976293954
Prometric Exam Collection 2019 01‫ودالشمال‬

.88
A nurse is caring for a group of newborns, which of the newborn would
need immediate attention?

Newborn with hyperthermic

.89
The nurse is assessing a baby with raised bruised area on the back left
side of the scalp?

Encephalohematoma

.91
A 30 year old married female has dilatation and curettage as
therapeutic abortion. A nurse was preparing for discharge instruction?
a. Take high protein diet
b. Use tampons during swimming
c. Avoid sexual intercourse for two weeks
d. Continue on bed rest for two weeks at home

.90
The nurse is caring for a patient 6 hours postpartum. Mother is Rh- and
baby Rh+. The doctor orders indirect Cohms test. Why?

It test for antibodies against Rh in mother

.92
Intracranial pressure

Elevated intracranial pressure (ICP) is considered a neurological emergency

.93
A 42 year old patient was to be prepared for the upper gastrointestinal
endoscopy. The patient anxiously asked the nurse about what will
happen in the procedure. The nurse explained that the endoscopy tube
will be inserted down his throat which will make him severely gag but

‫ مناقشة امتحانات البرومتريك للتمريض‬.. ‫قروب طور ذاتك‬ M- page 38


https://www.facebook.com/groups/1639727976293954
Prometric Exam Collection 2019 01‫ودالشمال‬

he will have to swallow it. Which type of communication the endoscopy


nurse has used here?

a. Assertive

b. Interactive

c. Therapeutic

d. Non-therapeutic

.94
Immediate complication for infant of diabetic mother

Hypoglycaemia

.95
Fetal complications of GDM >>>>>>> Macrosomia

 Neonatal hypoglycaemia
 Birth Weight
 Infants of women with GDM are at an increased risk of becoming
overweight or obese at a young age (during adolescence)
 Congenital malformations
 Macrosomia
Macrosomia

Macrosomia is a term that describes a baby who is born much larger


than average for their gestational age, which is the number of weeks in
the uterus. Babies with Macrosomia weigh over 8 pounds, 13 ounces.

 Causes of this condition include:


 diabetes in the mother

 obesity in the mother

 Genetics

 a medical condition in the baby

 Complications

‫ مناقشة امتحانات البرومتريك للتمريض‬.. ‫قروب طور ذاتك‬ M- page 39


https://www.facebook.com/groups/1639727976293954
Prometric Exam Collection 2019 01‫ودالشمال‬

For mother:
 Injury to the vagina.

 Bleeding after delivery.

 Uterine rupture.

For child
 Obesity.

 Abnormal blood sugar

.96
Handicap woman use wheelchair and other person not respect him and
she complains for the nurse what the nurse action?
a. Rehabilitation
b. Improve self esteem
.97
Women have nine childs, three of them have congenital anomalies and
one with Down syndrome, and she is poor socio social economic and she
did not use contraceptive method, the nurse should use any method of
counselling to explain important of contraceptive method type?
a. Individual counselling
b. Group counselling
c. No need to contraceptive method
.98
Hospital use research and new study this considers?
a. Hospital view
b. Hospital mission
c. Hospital policy
.99
Which of the following amounts of water per day should be ingested by
the average person to maintain hydration?
a. 2500cc
b. 1000cc
c. 1200cc
d. 500cc

‫ مناقشة امتحانات البرومتريك للتمريض‬.. ‫قروب طور ذاتك‬ M- page 40


https://www.facebook.com/groups/1639727976293954
Prometric Exam Collection 2019 01‫ودالشمال‬

.011
Droplet isolation the nurse how to use standard percussion PPE?

Mask _ gown_ eye protection _gloves

Putting on: Gown – Mask - eye protection - gloves

Removing:

‫ مناقشة امتحانات البرومتريك للتمريض‬.. ‫قروب طور ذاتك‬ M- page 41


https://www.facebook.com/groups/1639727976293954
Prometric Exam Collection 2019 01‫ودالشمال‬

.010
Lochia normally disappears after how many days postpartum?

a. 5 days
b. 7-10 days
c. 18-21 days
d. 28-30 days
Normally, lochia disappears after 10 days postpartum. What’s important to
remember is that the color of lochia gets to be lighter (from reddish to
whitish) and scantier every day.

.012
Nursing diagnosis is:

a. Puts the nursing care plan into action


b. A statement that describes potential health problem only
c. A statement that describes actual and potential health problems.
d. A statement that describes actual health problem only.
.013
Following a sigmoid scope, the patient should be observed for signs of
haemorrhage and:

a. nausea
b. perforation
c. flatus
d. fluid loss
.014
After an Rh (-) mother has delivered her Rh (+) baby, the mother is
given RhoGam. This is done in order to:

a. Prevent the recurrence of Rh(+) baby in future pregnancies

b. Prevent the mother from producing antibodies against the Rh(+)


antigen that she may have gotten when she delivered to her Rh(+)
baby

c. Ensure that future pregnancies will not lead to maternal illness

‫ مناقشة امتحانات البرومتريك للتمريض‬.. ‫قروب طور ذاتك‬ M- page 42


https://www.facebook.com/groups/1639727976293954
Prometric Exam Collection 2019 01‫ودالشمال‬

d. To prevent the newborn from having problems of incompatibility when it


breastfeeds

In Rh incompatibility, an Rh(-) mother will produce antibodies against the


fetal Rh (+) antigen which she may have gotten because of the mixing of
maternal and fetal blood during labor and delivery. Giving her RhoGam
right after birth will prevent her immune system from being permanently
sensitized to Rh antigen

.015
The baby has bluish legs and the HR is less than 100 b/m, the cry is
weak, full respiration, and good movement. What number of Apgar
score?

a. 9
b. 8
c. 7
d. 5
.016
Gestational diabetes is high blood sugar (diabetes) that starts or is
found during pregnancy. The best time for glucose screening tests
during pregnancy is:

a. 7 – 12 weeks
b. 17 – 18 weeks
c. 24 – 28 weeks
d. 20 – 35 weeks

 This test checks for gestational diabetes


 Most pregnant women have a glucose screening test between 24 and
28 weeks of pregnancy.
 The test may be done earlier women have a high glucose level in the
urine during routine prenatal visits, or have a high risk for diabetes
.017
The lochia on the first few days after delivery is characterized as

a. Pinkish with some blood clots

‫ مناقشة امتحانات البرومتريك للتمريض‬.. ‫قروب طور ذاتك‬ M- page 43


https://www.facebook.com/groups/1639727976293954
Prometric Exam Collection 2019 01‫ودالشمال‬

b. Whitish with some mucus


c. Reddish with some mucus
d. Serous with some brown tinged mucus
Reddish with some mucus right after delivery, the vaginal discharge called
lochia will be reddish because there is some blood, endometrial tissue and
mucus. Since it is not pure blood it is non-clotting

.018
Child have pharyngitis should be monitored to

Dehydration

.019
Nurse management role

 Staff management

 Case management

 Treatment planning

 Recruitment

 Scheduling

 Discharge planning

 Mentoring

 Developing educational plans

 Records management

‫ مناقشة امتحانات البرومتريك للتمريض‬.. ‫قروب طور ذاتك‬ M- page 44


https://www.facebook.com/groups/1639727976293954
Prometric Exam Collection 2019 01‫ودالشمال‬

.001
Nurse leadership and motivation
Leaders need to be able to motivate and inspire others
An effective leadership is taking into account such as the expectations and
motivations of their behaviour as well as the conditions within the
organization
.000
A pregnant woman comes to gynaecological and fetal movement
diminished, she need to:

Cardiotocogram (CTG) and ultrasound is advised.

This is a test which assesses the presence of fetal movements as well as the
presence of appropriate heart rate responses to the movement

.002
Symptoms of kwashiorkor - change in skin and hair color

 Change in skin and hair color (to a rust color) and texture

 fatigue and loss of muscle mass

 Diarrhea

 failure to grow or gain weight

 edema (swelling) of the ankles, feet, and belly

 damaged immune system, which can lead to more frequent and severe
infections

 irritability

 shock

.003
A pregnant woman in her 12 weeks comes to gynaecological with
vaginal bleeding, the diagnosis considers:

a. Placenta Previa

‫ مناقشة امتحانات البرومتريك للتمريض‬.. ‫قروب طور ذاتك‬ M- page 45


https://www.facebook.com/groups/1639727976293954
Prometric Exam Collection 2019 01‫ودالشمال‬

b. Abruptio placenta
c. Ectopic pregnancy
During the first 12 weeks of pregnancy, vaginal bleeding can be a sign of
miscarriage or ectopic pregnancy. However, many women who bleed at this
stage of pregnancy go on to have normal and successful pregnancies

.004
Patient has nausea, vomiting and muscle weakness comes to ER, after
assessment the doctor consider that sign and symptom due to:

a. Hypokalaemia
b. Hypernatremia
c. Hypokalaemia
d. Hypoglycaemia
Symptoms of hypokalaemia may include attacks of severe muscle weakness,
eventually leading to paralysis and possibly respiratory failure. Other
symptoms may include loss of appetite, nausea and vomiting

.005
A psychotic patient he is life alone and he fair and anxious what nursing
advice?

Coping with others

.006
NG tube for feeding should be removed when the patient become

Hungry or dehydrated

.007
Infant with Ventricular Septal Defects what is the main cause for this
problem?

a. Vitamin A deficiency
b. Vitamin c deviancy
c. Folic acid deficiency

‫ مناقشة امتحانات البرومتريك للتمريض‬.. ‫قروب طور ذاتك‬ M- page 46


https://www.facebook.com/groups/1639727976293954
Prometric Exam Collection 2019 01‫ودالشمال‬

.008
Patient with bronchitis have excess thick secretions what is the nursing
first responsibility?
a. Increase fluid intake
b. Administer oxygen
.009
The reflex that neonate quickly reaches their arms up, outwards and
cry the consider which of the following reflexes?
a. Moro reflex
b. Babinski reflex
c. Grasping reflex
d. Tonic neck reflex

The Moro reflex, also known as the ―startle reflex,‖ is a sudden, involuntary
reaction that babies have when their body feels unsupported or when they
are startled by a loud noise or sudden movement. This reaction causes babies
to unsteadily and quickly reach their arms up and outwards, like they are
reaching for something.

‫ مناقشة امتحانات البرومتريك للتمريض‬.. ‫قروب طور ذاتك‬ M- page 47


https://www.facebook.com/groups/1639727976293954
Prometric Exam Collection 2019 01‫ودالشمال‬

.021
Disease spread in a certain area? Endemic disease
An endemic disease is a disease that is always present in a certain population
or region. One of the most talked about endemic diseases is malaria
.020
Patient in operation express heart arrest how many compression should
do per minute?
a. 15
b. 30
c. 50
d. 100
Push hard at a rate of 100 to 120 compressions a minute, after 30 chest
compressions at a rate of 100 to 120 a minute, give 2 rescue breaths

.022
Pregnant patient admission to triage with abdominal pain on
assessment the FHR found 70 p/m. what is the first action for the nurse?
a. Call the doctor
b. Report the finding
c. Put the patient in left lateral position

Fetal heart rate decelerations occurred in the supine position, with a drop in
maternal BP from 120/77 (BP,) to 65/39 mmHg (BP,), and in fetal oxygen
saturation from 42% to 22%. When left lateral position was adopted again,
the decelerations disappeared and oxygen saturation increased to 48%.

.023
Pregnant women in the first stage of labor and has progressed from 3 to
7 cm in cervical dilation. In which of the following phases the cervical
dilation occurs most rapidly?
a. Preparatory phase
b. Latent phase
c. Active phase
d. Transition phase

‫ مناقشة امتحانات البرومتريك للتمريض‬.. ‫قروب طور ذاتك‬ M- page 48


https://www.facebook.com/groups/1639727976293954
Prometric Exam Collection 2019 01‫ودالشمال‬

.024
A pregnant client is making her first Antepartum visit. She has a two
year old son born at 40 weeks. a 5 year old daughter born at 38 weeks
and 7 year old twin daughters born at 35 weeks. She had a spontaneous
abortion 3 years ago at 10 weeks. Using the GTPAL format the nurse
should identify that the client is:
a. G4 T3 P2 A1 L4
b. G5 T2 P2 A1 L4
c. C. G5 T2 P1 A1 L4
d. G4 T3 P1 A1 L4
5 pregnancies, one abortion and 4 living children

.025
After insertion the NG to patient what is the priority action before
starting the feeding?

a. Aspirate for stomach secretion


b. x-ray
.026
Patient receiving TPN what is the first responsibility to check

a. hypoglycaemia
b. Hypokalaemia
c. hyperglycaemia
.027
Patient arrived to ER with chest pain that recently move to the jaw and
shoulder and suspected with MI what is change in ECG support that
diagnosis?

a. Absent of p wave
b. Elevated ST
.028
Ringer lactate solution is?

a. Isotonic
b. Hypotonic
c. Hypertonic

‫ مناقشة امتحانات البرومتريك للتمريض‬.. ‫قروب طور ذاتك‬ M- page 49


https://www.facebook.com/groups/1639727976293954
Prometric Exam Collection 2019 01‫ودالشمال‬

.029
The most important procedure that prevents cross contamination and
infection

a. Cleaning
b. Disinfecting
c. Sterilizing
d. Hand washing

‫ مناقشة امتحانات البرومتريك للتمريض‬.. ‫قروب طور ذاتك‬ M- page 50


https://www.facebook.com/groups/1639727976293954
Prometric Exam Collection 2019 01‫ودالشمال‬

.031
Patient post liver biopsy is lie on the right side, why?

Prevent from haemorrhage

.030
The highest cause of post-partum haemorrhage is?

a. Retained placenta
b. Placenta Previa
c. Cord prolapse
.032
19 Years old female patient arrive emergency department with vomiting and
diarrhoea and she is lethargic, when ask her fir name and place she answered
appropriate and she knows she is in hospital, where you will check
temperature correctly

a. Axillary
b. Oral
c. Rectal
d. Tympani
.033
Mode of transmission of diphtheria disease >>>>> droplets

 Diphtheria is an infection caused by the Corynebacterium


diphtheriae bacterium.
 Spreads (transmits) from person to person, usually through respiratory
droplets, like from coughing or sneezing

.034
Woman complains of brown vaginal discharge in after 10 days of
childbirth the woman needs more education about:

I think know more about the normal lochia and vaginal discharges

‫ مناقشة امتحانات البرومتريك للتمريض‬.. ‫قروب طور ذاتك‬ M- page 51


https://www.facebook.com/groups/1639727976293954
Prometric Exam Collection 2019 01‫ودالشمال‬

.035
Delusion of grandeur

Is the fixed, false belief that one possesses superior qualities such as genius,
fame, omnipotence, or wealth. It is most often a symptom of schizophrenia,
but can also be a symptom found in psychotic or bipolar disorders, as well as
dementia (such as Alzheimer's)

.036
A pregnant woman knows her fetus is girl after ultra sound and decided
to do Abortion because the parents did not want girl. What the nurse
do?

a. Communicate with family and follow religious aspects


b. Mother has right for abortion
.037
25 years old female after labour few days came to gynaecological
complain of vaginal bleeding she changed 2 pads with heavy blood
what’s the best action for her condition:

a. Hormonal therapy
b. Rubs the funds to enhance Uterus contractions
c. Assesses and monitor vital signs

Severe uterine bleeding is usually treated on an emergency basis with a short


course of high-dose estrogen therapy

.038
Postpartum nurse is preparing to care for a woman who has just
delivered a healthy newborn infant. In the immediate postpartum
period the nurse plans to take the woman’s vital signs
a. Every 30 minutes during the first hour and then every hour for the next
two hours
b. Every 15 minutes during the first hour and then every 30 minutes for
the next two hours

‫ مناقشة امتحانات البرومتريك للتمريض‬.. ‫قروب طور ذاتك‬ M- page 52


https://www.facebook.com/groups/1639727976293954
Prometric Exam Collection 2019 01‫ودالشمال‬

c. Every hour for the first 2 hours and then every 4 hours
d. Every 5 minutes for the first 30 minutes and then every hour for the next
4 hours

.039
In acute appendicitis patient, the rebound tenderness found in any part of the
body? Right lower quadrant

.041
What is affected organs in Gastroesophageal reflux disease?

a. Stomach and oesophagus


b. Liver and spleen
c. Liver and bile duct
d. Stomach and small intestine

Gastroesophageal reflux disease, or GERD, is a digestive disorder that


affects the lower oesophageal sphincter (LES), the ring of muscle between
the oesophagus and stomach. Many people, including pregnant women,
suffer from heartburn or acid indigestion caused by GERD

‫ مناقشة امتحانات البرومتريك للتمريض‬.. ‫قروب طور ذاتك‬ M- page 53


https://www.facebook.com/groups/1639727976293954
Prometric Exam Collection 2019 01‫ودالشمال‬

.040
Pregnant woman 34 weeks come to gynaecological with high blood
pressure, this elevation during pregnancy called >>>> pregnancy-induced
hypertension PIH

Also called preeclampsia, toxemia, toxemia of pregnancy or gestational


hypertension

.042
A nurse is providing instructions to a mother who has been diagnosed
with mastitis. Which of the following statements if made by the mother
indicates a need for further teaching?

a. ―I need to take antibiotics. And I should begin to feel better in 24-48


hours.‖

b. ―I can use analgesics to assist in alleviating some of the discomfort.‖

c. ―I need to wear a supportive bra to relieve the discomfort.‖

d. “I need to stop breastfeeding until this condition resolves.”

In most cases, the mother can continue to breastfeed with both breasts. If the
affected breast is too sore the mother can pump the breast gently. Regular
emptying of the breast is important to prevent abscess formation

.043
A 69-year-old female patient in the Medical Ward is in a semi-conscious
state. Her pancreatic cancer is metastasized to her liver and lungs and
she is admitted for supportive treatment. Her physicians discussed with
the family that she will not be given the CPR to save the life are she goes
into the cardiac arrest. Her two sons agreed but the daughter is
indecisive. What is the critical thinking behind not providing
Cardiopulmonary Resuscitation?

a. Unilateral judgment of health professionals

b. Refusal of patient’s right to treatment

‫ مناقشة امتحانات البرومتريك للتمريض‬.. ‫قروب طور ذاتك‬ M- page 54


https://www.facebook.com/groups/1639727976293954
Prometric Exam Collection 2019 01‫ودالشمال‬

c. Ethical dilemma and indecisiveness

d. Mercy killing to ease suffering

.044
28 year old female, who is 33 weeks pregnant with her second child, has
uncontrolled hypertension. What risk factor below found in the
patient's health history places her at risk for Abruptio placentae?

a. childhood polio
b. preeclampsia
c. C-section
d. her age
.045
While caring for a neonate with a meningococcal, the nurse should
AVOID positioning the child on the:

a. Abdomen
b. Left side
c. Right side
d. Back
.046
A pregnant woman at 32 weeks' gestation complains of feeling dizzy and
lightheaded while her fundal height is being measured. Her skin is pale
and moist. The nurse's initial response would be to:

a. Assess the woman’s blood pressure and pulse


b. Have the woman breathe into a paper bag
c. Raise the woman’s legs
d. Turn the woman on her left side.
During a fundal height measurement, the woman is placed in a supine
position. This woman is experiencing supine hypotension as a result of
uterine compression of the vena cava and abdominal aorta. Turning her on
her side will remove the compression and restore cardiac output and blood
pressure.

‫ مناقشة امتحانات البرومتريك للتمريض‬.. ‫قروب طور ذاتك‬ M- page 55


https://www.facebook.com/groups/1639727976293954
Prometric Exam Collection 2019 01‫ودالشمال‬

.047
Which of the following is described as premature separation of a normally
implanted placenta during the second half of pregnancy with severe
haemorrhage occurrence?

a. Placenta Previa

b. Ectopic pregnancy

c. Incompetent cervix

d. Abruptio placentae

.048
A 6 year old male is diagnosed with Nephrotic syndrome. When nursing
care for the patient the plan of diet should be?

a. high salt, High fat


b. high salt, High cholesterol
c. low salt, low fat
d. low protein, high salt
.049
Patient diagnosed with Nephrotic syndrome, which of the following
treatment used for patient? Corticosteroids

Diagnosis
 Urine tests
 Blood tests
 Sample of kidney tissue for testing
Treatment
 Blood pressure medications: angiotensin-converting enzyme (ACE):
 Benazepril (Lotensin)
 Captopril
 Enalapril (Vasotec)
 Water pills(diuretics) :
 Furosemide (Lasix)
 Hydrochlorothiazide
 Cholesterol-reducing medications

‫ مناقشة امتحانات البرومتريك للتمريض‬.. ‫قروب طور ذاتك‬ M- page 56


https://www.facebook.com/groups/1639727976293954
Prometric Exam Collection 2019 01‫ودالشمال‬

 Atorvastatin (Lipitor)
 Fluvastatin (Lescol)
 Rosuvastatin (Crestor)
 Simvastatin (Zocor)
 Blood thinners.
 Heparin
 Warfarin
 Immune system-suppressing medications:
Medications to control the immune system, such as corticosteroids, may
decrease the inflammation that accompanies underlying conditions

.051
What are the more vitamins that promote wound healing? Vitamin C

Vitamin C, also known as ascorbic acid, is required for the synthesis of


collagen. It is also a highly effective antioxidant protecting cells from
damage by free radicals.

.050
When involved in prenatal teaching. The nurse should advise the clients
that an increase in vaginal secretions during pregnancy is called
leucorrhoea and is caused by increased:

a. Metabolic rates
b. Production of estrogen
c. Functioning of the Bartholin glands
d. Supply of sodium chloride to the cells of the vagina
The increase of estrogen during pregnancy causes hyperplasia of the vaginal
mucosa. Which leads to increased production of mucus by the endocervical
glands? The mucus contains exfoliated epithelial cells.

.052
How you confirm if the Psychiatric patient taking his medications or
not?

Forward and backward moving of jaw

‫ مناقشة امتحانات البرومتريك للتمريض‬.. ‫قروب طور ذاتك‬ M- page 57


https://www.facebook.com/groups/1639727976293954
Prometric Exam Collection 2019 01‫ودالشمال‬

.053
Which of the following terms applies to the tiny, Blanched. Slightly
raised end arterioles found on the face, Neck, Arms and chest during
pregnancy?

a. Epulis
b. Linea nigra
c. Striae gravidarum
d. Telangiectasia

The dilated arterioles that occur during pregnancy are due to the elevated
level of circulating estrogen.

.054
Democratic definition

Democratic leadership, also known as participative leadership or


shared leadership is a type of leadership style in which members of the
group take a more participative role in the decision-making process.
This type of leadership can apply to any organization

.055
Cystic fibrosis

Cystic fibrosis (CF) is a life-threatening disease that causes thick, sticky


mucus to build up in the lungs and digestive tract. People with CF need to
eat foods that are high in calories and protein throughout the day

Diet

High in calories and protein

.056
Parkinson disease have a hard time swallowing because they lose
control of their mouth and throat muscles As a nurse which of the
following is a Parkinson diet ?

a. Solid

‫ مناقشة امتحانات البرومتريك للتمريض‬.. ‫قروب طور ذاتك‬ M- page 58


https://www.facebook.com/groups/1639727976293954
Prometric Exam Collection 2019 01‫ودالشمال‬

b. Semi solid

c. Liquid

d. Clear liquid

.057
A nurse is assessing a client in the 4th stage if labour and notes that the
fundus is firm but that bleeding is excessive. The initial nursing action
would be which of the following?

a. Massage the fundus


b. Place the mother in the trendelenburg’s position
c. Notify the physician
d. Record the findings
If the bleeding is excessive the cause may be laceration of the cervix or birth
canal. Massaging the fundus if it is firm will not assist in controlling the
bleeding. Trendelenburg’s position is to be avoided because it may interfere
with cardiac function.

.058
Which of the following would the nurse most likely expect to find when
assessing a pregnant client with abruption placenta?

a. Excessive vaginal bleeding


b. Rigid. board-like abdomen
c. Titanic uterine contractions
d. Premature rupture of membranes
The most common assessment finding in a client with abruption placenta is a
rigid or board like abdomen. Pain usually reported as a sharp stabbing
sensation high in the uterine fundus with the initial separation also is
common.

.059
Discharge teaching for a child with celiac disease would include
instructions about avoiding which of the following?

a. Rice

‫ مناقشة امتحانات البرومتريك للتمريض‬.. ‫قروب طور ذاتك‬ M- page 59


https://www.facebook.com/groups/1639727976293954
Prometric Exam Collection 2019 01‫ودالشمال‬

b. Milk
c. Wheat
d. Chicken
Children with celiac disease cannot tolerate or digest gluten. Therefore,
because of its gluten content wheat and wheat-containing products must be
avoided.
.061
As per of a neurological assessment, which of the following is associated
with the higher score on the Glasgow coma scale?

a.Eye opening to pain, no verbalization


b.Confused, obey commands
c.Localized pain, abnormal extension
d.Eye opening to speech, confused
. .060
During the assessment the client tells the nurse he has pain in the left
Knee. This information is considered:

a. Objective data
b. Subjective data
c. No relevant data
d. Historical data
.062
Which of the following is an example of epidemic point source?

a. Tuberculosis
b. Public health agency
c. Contaminated water source
d. Communicable disease pavilion

.063

A female patient admitted for right lower abdominal pain, nausea,


vomiting, and constipation, the patient has rebound tenderness in right
lower guardant, indication for:

‫ مناقشة امتحانات البرومتريك للتمريض‬.. ‫قروب طور ذاتك‬ M- page 60


https://www.facebook.com/groups/1639727976293954
Prometric Exam Collection 2019 01‫ودالشمال‬

a. Appendicitis

b. Ulcerative colitis

c. Pancreatitis

d. Abdominal distension

.064
A most important nursing measure in the prevention of
thrombophlebitis for the postpartum patient is:
a. Isometric exercise
b. Early ambulation
c. Anticoagulant
d. Elastic stocking
.065
The principle of fairness is known as:
a. Advocacy
b. Autonomy
e. Justice
c. Accountability
.066
Pulse pressure is defined as which of the following:
a. Difference between venous and systolic pressure
b. Expansion of the artery as blood moves through it.
c. Difference between arterial and venous pressure.
d. Difference between systolic and diastolic pressure.
.067
The ability to answer for your actions is known as:
a. Confidentiality
b. Accountability
c. Advocacy
d. Veracity
.068
The most important stage of the nursing process is:
a. Evaluation

‫ مناقشة امتحانات البرومتريك للتمريض‬.. ‫قروب طور ذاتك‬ M- page 61


https://www.facebook.com/groups/1639727976293954
Prometric Exam Collection 2019 01‫ودالشمال‬

b. Assessment
c. Planning
d. Implementation
.069
The duty to protect privileged information is called:
a. Ethics
b. Morals
c. Confidentiality
d. Honesty
.071
Informed consent is a method that promotes:
a. Confidence
b. Justice
c. Autonomy
d. Veracity
.070
To respect a patient's personal space, the nurse:
a. Avoids communication
b. Ensures the curtains are pulled around the patient's bed
c. Stands 2 metres away from the bed
d. Explains nursing care and procedures
.072
The Code of Professional Conduct is produced and published by:
a. The Nursing and Midwifery Council
b. The Royal College of Physicians
c. The Royal College of Nursing
d. The Department of Health
.073
The nurse acts as an advocate in order to:
a. Get their point across
b. Offer confidentiality
c. Asses the patient's point of view and be prepared to articulate this
d. Abide by their contract of employment

‫ مناقشة امتحانات البرومتريك للتمريض‬.. ‫قروب طور ذاتك‬ M- page 62


https://www.facebook.com/groups/1639727976293954
Prometric Exam Collection 2019 01‫ودالشمال‬

.074
Patient with blood infusion, you found it with fever after the infusion
begins with 19 minutes. The right action is:
a. Call the doctor.
b. Continue blood infusion with slow rate.
c. Stop infusion immediately.
d. a+b
.075
The most reliable method used for sterilizing hospital equipment to be
free of spores and bacteria is:
a. Boiling the equipment
b. Washing and drying it thoroughly after use
c. Applying steam under pressure in an autoclave
d. d. Soaking in strong chemical
.076
Elsa Santos is a 18 year old student admitted to the ward with a
diagnosis of epilepsy. She tells the nurse that she is experiencing a
generalized tingling sensation and “smells roses”. The nurse
understands that Esla is probably experiencing:
a. An olfactory hallucination
b. An acute CVA
c. An aura
d. An acute alcohol withdrawal
.077
Asking the patient if he is allergic to any medication is a part of the
……………phase of the nursing process?
a. Evaluation
b. Planning
c. Implementation
d. Assessment
.078
Dorsal recumbent position is used when performing the following
procedures EXCEPT:
a. Vaginal examination

‫ مناقشة امتحانات البرومتريك للتمريض‬.. ‫قروب طور ذاتك‬ M- page 63


https://www.facebook.com/groups/1639727976293954
Prometric Exam Collection 2019 01‫ودالشمال‬

b. Cystoscopic examination
c. Urinary catheter insertion
d. Suppository insertion
.079
The force with which the blood is pushing against the arterial walls
when the ventricles are contracting is called:
a. Diastolic pressure
b. Pressure gradient
c. Systolic pressure
d. Pulse pressure
.081
To collect urine specimen, the most accurate method:
a. Mid-stream urine
b. Provide sterile bedpan and collect urine
c. Catheterize the patient
.080
Reactions of penicillin:
a. Vomiting
b. Nausea.
c. Anaphylactic shock
.082
A 40 year old male patient is complaining of chronic progressive and
mental deterioration is admitted to the unit. The nurse recognizes that
these characteristics indicate a disease that results in degeneration of
the basal ganglia and cerebral cortex. The disease is called:
a. Guillain-Barre syndrome
b. Myasthenia gravis
c. Huntington disease
d. Multiple sclerosis
.083
Dyspnea is defined as:
a. Difficult respiration
b. Absence of breathing
c. Cyanosis Pallor

‫ مناقشة امتحانات البرومتريك للتمريض‬.. ‫قروب طور ذاتك‬ M- page 64


https://www.facebook.com/groups/1639727976293954
Prometric Exam Collection 2019 01‫ودالشمال‬

.084
All of the following is used Central Venous Catheter, EXCEPT
a. Difficult to insert peripheral catheter
b. IV medications are irritating to peripheral veins
c. Clients require short term IV medication
d. Clients require long term IV medication
.085
In acute stage of osteomyelitis, the child should be:
a. Up and about
b. Up with crutches.
c. Up in a wheelchair
d. Confined to bed
.086
Nursing care of a child admitted with acute glomerulonephritis, should
be directed toward:
a. eliminating sodium from diet
b. promoting diuresis
c. enforcing strict bed rest
d. forcing fluids
.087
At which age is administration of the measles vaccine the most effective:
a. 15 months
b. 6 months
c. 12 months
d. 2 months
.088
The cure for chronic renal failure is:
a. force fluids
b. kidney transplant
c. diuretics
d. dialysis
.089
Which of the following complication is life threatening after a
penetrating abdominal injury?

‫ مناقشة امتحانات البرومتريك للتمريض‬.. ‫قروب طور ذاتك‬ M- page 65


https://www.facebook.com/groups/1639727976293954
Prometric Exam Collection 2019 01‫ودالشمال‬

a. Peritonitis
b. Abdominal
c. distension
d. Haemorrhage
.091
In which of the following ways can ascorbic acid be administered?
a. IM
b. Diluted intravenous fluids
c. Orally
d. All of the above
.090
Within 48 hours of burns, management is focused on:
a. Hydration
b. Prevention of infection
c. Inflammation
.092
This of the following is a form of radiography:
a. Contrast medium
b. C.T. Scan
c. Fluoroscopy.
d. All of the above
.093
Mrs. Kelly tells the nurse that her arm is sore from an injection she
received early in the day she states “the nurse gave me a shot, and I
heard her say that the needle was a 25 gauge. Isn’t that too big for a
local injection?” Your best response would be:
a. 25 gauge needles is very small. So you are wrong about the pain from
the injection.
b. Really, Mrs. Kelly, no once gets hurts with a 25 gauge needle. you are
over reacting to the shot
c. 25 gauge needles are very small. You are just nervous about the
injection.
d. 25 gauge needles is a small needle, but it can cause some
discomfort. Let me see your arm.

‫ مناقشة امتحانات البرومتريك للتمريض‬.. ‫قروب طور ذاتك‬ M- page 66


https://www.facebook.com/groups/1639727976293954
Prometric Exam Collection 2019 01‫ودالشمال‬

.094

In palpating the client’s breast, which of the following position is


necessary for the patient to assume before the start of the procedure?
a. Lithotomy.
b. Dorsal recumbent
c. Sitting
d. Supine
.095
Pre-op medication is given:
a. as physician ordered
b. 45 min. before operation
c. previous night of operation
d. 2 hours before operation
.096
Immediately after a child is admitted with acute bacterial meningitis,
the nurse should plan to:
a. Restrict parental visiting until isolation is discontinued
b. Administer oral antibiotic medication as ordered
c. Check the child level of consciousness every hour
d. Assess the child vital signs every 3 hours

.097
The intentional wound is:
a. The tissue is traumatized without a break in the skin.
b. Occur accidentally.
c. The mucous membrane or skin surface is broken.
d. Occur during therapy.
.098

Patient with Nephrotic syndrome, which of the following used for treating
the patient? Corticosteroids

Diagnosis
 Urine tests

‫ مناقشة امتحانات البرومتريك للتمريض‬.. ‫قروب طور ذاتك‬ M- page 67


https://www.facebook.com/groups/1639727976293954
Prometric Exam Collection 2019 01‫ودالشمال‬

 Blood tests
 Sample of kidney tissue for testing
Treatment
 Blood pressure medications: angiotensin-converting enzyme (ACE):
 Benazepril (Lotensin)
 Captopril
 Enalapril (Vasotec)
 Water pills (diuretics) :
 Furosemide (Lasix)
 Hydrochlorothiazide
 Cholesterol-reducing medications
 Atorvastatin (Lipitor)
 Fluvastatin (Lescol)
 Rosuvastatin (Crestor)
 Simvastatin (Zocor)
 Blood thinners.
 Heparin
 Warfarin
 Immune system-suppressing medications:
Medications to control the immune system, such as corticosteroids, may
decrease the inflammation that accompanies underlying conditions

.099
Patient with parkinson’s disease have a hard time swallowing because they
lose control of their mouth and throat muscles As a nurse which of the
following is a Parkinson’s diet ?

a. Solid
b. Semi solid
c. Liquid
d. Clear liquid
.211
AP wave with increased amplitude can indicate hypokalaemia.

A P wave with decreased amplitude can indicate hyperkalaemia.

‫ مناقشة امتحانات البرومتريك للتمريض‬.. ‫قروب طور ذاتك‬ M- page 68


https://www.facebook.com/groups/1639727976293954
Prometric Exam Collection 2019 01‫ودالشمال‬

‫ مناقشة امتحانات البرومتريك للتمريض‬.. ‫قروب طور ذاتك‬ M- page 69


https://www.facebook.com/groups/1639727976293954
Prometric Exam Collection 2019 01‫ودالشمال‬

‫ مناقشة امتحانات البرومتريك للتمريض‬.. ‫قروب طور ذاتك‬ M- page 70


https://www.facebook.com/groups/1639727976293954
Prometric Exam Collection 2019 01‫ودالشمال‬

.210
A nurse is preparing to assess the uterine fundus of a client in the immediate
postpartum period. When the nurse locates the fundus she notes that the
uterus feels soft and boggy. Which of the following nursing interventions
would be most appropriate initially?
a. Massage the fundus until it is firm
b. Elevate the mother’s legs
c. Push on the uterus to assist in expressing clots
d. Encourage the mother to void

‫ مناقشة امتحانات البرومتريك للتمريض‬.. ‫قروب طور ذاتك‬ M- page 71


https://www.facebook.com/groups/1639727976293954
Prometric Exam Collection 2019 01‫ودالشمال‬

.212
Site for Lumber Puncture
A lumbar puncture (LP), also called a spinal tap, is an invasive outpatient
procedure used to remove a sample of cerebrospinal fluid (CSF) from the
subarachnoid space in the spine.

A spinal needle is inserted between the lumbar vertebrae L3/L4, L4/L5 or


L5/S1

.213
A 22-year-old patient is admitted in the male Medical diagnosis of
tonsillar abscess. He has high fever and severs along with dysphagia,
difficulty in talking and in opening patient is planned for needle
aspiration of the abscess intravenous antibiotics including penicillin.
What expected outcome of nursing care should be prioritized?
a. Patient is comfortable and has minimum pain
b. Patient is able to communicate appropriately
c. Able to swallow fluids and soft diet more easily
d. Normal body temperature and stable vital signs

‫ مناقشة امتحانات البرومتريك للتمريض‬.. ‫قروب طور ذاتك‬ M- page 72


https://www.facebook.com/groups/1639727976293954
Prometric Exam Collection 2019 01‫ودالشمال‬

.214

Vaccination in 2,6,12 months

.215
A pregnant women in fourth month comes to gynaecological with
MRSA, after assessment the doctor order put him in isolation, what
type of isolation

a. Air borne
b. Contact
c. Droplet

‫ مناقشة امتحانات البرومتريك للتمريض‬.. ‫قروب طور ذاتك‬ M- page 73


https://www.facebook.com/groups/1639727976293954
Prometric Exam Collection 2019 01‫ودالشمال‬

.216
You have just admitted a patient with bacterial meningitis to the
medical-surgical unit. The patient complains of a severe headache with
photophobia and has a temperature of 102.60 F orally. Which type of
isolation should be suitable for the patient?

a. Air borne
b. Contact
c. Droplet
.217
The nurse instructing mother how to feeding her baby with cleft lip,
what statement indicate the mother understanding?

I positioned my baby upright position when feeding

.218
What the meaning of term MMR?

MMR is the combined vaccine that protects against measles, mumps and
rubella

‫ مناقشة امتحانات البرومتريك للتمريض‬.. ‫قروب طور ذاتك‬ M- page 74


https://www.facebook.com/groups/1639727976293954
Prometric Exam Collection 2019 01‫ودالشمال‬

.219
Preterm infant delivered at 28 gestational age by weight 850 mg and
doctor order refers infant to NICU, when the nurse receiving she
document the infant:

a. Low gestational age


b. Normal gestational age
c. Very Low gestational age

.201
Pregnant women in labour are taking induction of labour, CTG shown
normal fetal heart rate, the dilatation of cervix 6cm, and the contraction
regular every 2 min in duration 80sec, what the nurse do?

a. Stop induction
b. Order to physician
c. Decrease dose
d. Document the result
.200
A patient with seizures, what is the best intervention for nursing during
convulsion:

a. Patient safety.
b. put mouth gag to the patient
c. Give Medication

‫ مناقشة امتحانات البرومتريك للتمريض‬.. ‫قروب طور ذاتك‬ M- page 75


https://www.facebook.com/groups/1639727976293954
Prometric Exam Collection 2019 01‫ودالشمال‬

.202
Psychotic patient and the doctor order antidepressants treatment, the
patient asks the nurse how long it takes for the treatment action to
begin.

a. 3 – 4 days
b. 7 – 8 days
c. 10 – 14 days
d. 14 – 20 days

.203
When start patient discharge plan:

a. At admission
b. Before discharge days
c. When the doctor writes the order
d. After discharge
.204
A 40 year old man comes in the internal medicine unit, when assessment
the nurse found glass coma scale 10 and he appears sleeping, what the
nurse do first:

a. Elevate bed side rail


b. Awake him from sleeping
c. Order to physician
d. Give him his medicine and leave the room
.205
Respiratory syncytial virus (RSV) >>>>>> spread through
direct contact with the virus

Respiratory syncytial virus (RSV) is the most common cause of


bronchiolitis (inflammation of the small airways in the lung) and pneumonia
(infection of the lungs) in children younger than 1 year. It is also a
significant cause of respiratory illness in older adults.

‫ مناقشة امتحانات البرومتريك للتمريض‬.. ‫قروب طور ذاتك‬ M- page 76


https://www.facebook.com/groups/1639727976293954
Prometric Exam Collection 2019 01‫ودالشمال‬

.206
After baby circumcision done the injury become bleeding what the
nurse do? Apply pressure with sterile gauze

.207
Diabetes insipidus >>>>>> posterior pituitary

.208
Complication of hypothyroidism in pregnancy women>> postpartum
haemorrhage

Treatment with sufficient amounts of thyroid hormone replacement


significantly reduces the risk for developing pregnancy complications
associated with hypothyroidism, such as

 premature birth
 preeclampsia
 miscarriage
 postpartum haemorrhage
 anemia
 Abruptio placentae

.209
Which measure would be least effective in preventing postpartum
hemorrhage?

a. Administer Methergine 0.2 mg every 6 hours for 4 doses as ordered


b. Encourage the woman to void every 2 hours
c. Massage the fundus every hour for the first 24 hours following
birth
d. Teach the woman the importance of rest and nutrition to enhance
healing
Fundal massage, also called uterine massage, is a technique used to reduce
bleeding and cramping of the uterus after childbirth or after an abortion. As
the uterus returns to its non-pregnant size, its muscles contract strongly, this
can cause pain

‫ مناقشة امتحانات البرومتريك للتمريض‬.. ‫قروب طور ذاتك‬ M- page 77


https://www.facebook.com/groups/1639727976293954
Prometric Exam Collection 2019 01‫ودالشمال‬

.221
Women have very heavy bright red bleeding (rubra) immediately post-
delivery, what the nurse do?

a. Oder to doctor
b. Fundus massage
c. Talk patient it’s normal
.220
Second infertility causes >>>>>> men

Secondary infertility is the inability to become pregnant or to carry a baby to


term after previously giving birth to a baby

Possible causes of secondary infertility are:

 Impaired sperm production, function or delivery in men.


 Fallopian tube damage, ovulation disorders, endometriosis and uterine
conditions in women.

.222
22-year-old patient is admitted in the male diagnosis of tensile abscess.
He has high fever along with dysphasia, difficulty in talking and in
patient is planned for needle aspiration of the intravenous antibiotics
including penicillin. What the important nursing diagnosis for the
patient?
a. Imbalance nutrition due to inadequate
b. Acute pain related to throat inflammation
c. Impaired swallowing related to dysphagia
d. Hyperthermia related to acute infection

.223
The nurse visits a school of girls and educates him about breast self-
exam, which of the following statement indicates the student
understanding?

They are participate with nurse during discussion

‫ مناقشة امتحانات البرومتريك للتمريض‬.. ‫قروب طور ذاتك‬ M- page 78


https://www.facebook.com/groups/1639727976293954
Prometric Exam Collection 2019 01‫ودالشمال‬

.224
Mammogram>>>> early detection of breast cancer

Mammogram is an x-ray that allows a qualified specialist to examine the


breast tissue.

.225
The nurse teaching pregnant women about important come to hospital
when she has sign and symptom can lead to complication like?

Vaginal bleeding

.226
Which of the following patient can be transferee from emergency unit to
inpatient ward?

a. Patient with DKA has nausea and vomiting


b. Patient with respiratory distress
c. Patient has operation next day
d. Patient with hematoma on manitol
.227
Obese patient planned for sleeve gastrectomy, what the purpose from
the operation?

To reduce stomach size

‫ مناقشة امتحانات البرومتريك للتمريض‬.. ‫قروب طور ذاتك‬ M- page 79


https://www.facebook.com/groups/1639727976293954
Prometric Exam Collection 2019 01‫ودالشمال‬

.228
Patient have intracranial pressure, the doctor order to check vital signs
and sign and symptom of intracranial pressure every 15 minutes, which
of the following you are check it in the first?

a. If patient have severe headache


b. Temp
c. pulse
d. Type and characteristics of drainage

.229
Patient has oedema in leg what you do?

Elevate the leg

.231
In the 12th week of gestation, a client completely expels the products of
conception. Because the client is Rh-negative, the nurse must:

a. Administer RhoGam within 72 hours


b. Make certain she receives RhoGAM on her first clinic visit
c. Not give RhoGam since it is not the birth of a stillborn
d. Make certain the client does not receive RhoGAM since the gestation
only lasted 12 weeks
.230
A nurse is assessing a client in labor and finds that her contractions are
lasting 60 seconds every 4 minutes and that her cervix is 6 cm dilated.
The nurse would document that the client is in what phase of stage 1
labor?

a. Active phase
b. Early phase
c. Latent phase
d. Transitional phase

‫ مناقشة امتحانات البرومتريك للتمريض‬.. ‫قروب طور ذاتك‬ M- page 80


https://www.facebook.com/groups/1639727976293954
Prometric Exam Collection 2019 01‫ودالشمال‬

.232
During a client’s labor, the fetus’ head is at station +1. This indicates
that the presenting part is:

a. On the perineum
b. Slightly below the ischial spines
c. Slightly above the ischial spines

.233
Women have nine childs, three of them have congenital anomalies and
one with Down syndrome, and she is poor socio social economic and she
did not use contraceptive method, the nurse should use any method of
counselling to explain important of contraceptive method and type?

‫ مناقشة امتحانات البرومتريك للتمريض‬.. ‫قروب طور ذاتك‬ M- page 81


https://www.facebook.com/groups/1639727976293954
Prometric Exam Collection 2019 01‫ودالشمال‬

a. Individual counselling
b. Group counselling
c. No need to contraceptive method
.234
A client who is gravida 1, para 0 is admitted in labor. Her cervix is
100% effaced, and she is dilated to 3 cm. Her fetus is at +1 station. The
nurse is aware that the fetus’ head is:

a. Not yet engaged

b. Entering the pelvic inlet

c. Below the ischial spines

d. Visible at the vaginal opening

A station of +1 indicates that the fetal head is 1 cm below the ischial spines

.235
During the taking history from 9 year old boy he say to nurse he lives
with his father, mother and grandfather with 8 children in small house,
his father working and his monthly salary is simple and not sufficient to
their need, from the history the nurse know this family consider:

Low economic status


.236
Tuberculosis patient should be isolated in>>>> as negative-
pressure rooms

Limited number of Airborne Infection Isolation Rooms (AIRR)

Uses mask for this patient >>>> N95 Mask

.237
Tuberculosis patient in isolation room, who is wearing the mask when
contact with the patient?

a. The patient
b. Medical staff

‫ مناقشة امتحانات البرومتريك للتمريض‬.. ‫قروب طور ذاتك‬ M- page 82


https://www.facebook.com/groups/1639727976293954
Prometric Exam Collection 2019 01‫ودالشمال‬

c. His family when visit


d. Any one inter patient room
.238
Patient comes to ER with cough, weight loss, fever and night sweats,
after chest x – ray the doctor diagnosed the case tuberculosis and order
to give hypertonic solution, what the suitable should be given?

a. Dextrose 10%
b. Normal saline 9%
c. Normal saline 45%
d. Sodium chloride 5%

The following is a list of solutions by their tonicity:


Hypertonic:
o D5 NaCl
o D5 in Lactated ringers
o D5 0.45% NaCl
Isotonic:
o 0.9% NaCl (Normal Saline)
o Lactated Ringers
o D5W (In the bag)
Hypotonic:
o D5W (in the body)
o 0.25% NaCl
o 0.45% NaCl (half normal saline)
o 2.5% Dextrose

Why D5W is listed under two categories?


Well because it is a trickster. The actual solution in the bad is isotonic, but
once you give D5W to patients the body metabolizes the glucose molecules
that were once causing the solution to be isotonic. The solution is now
missing solute, causing it to become a hypotonic solution.

‫ مناقشة امتحانات البرومتريك للتمريض‬.. ‫قروب طور ذاتك‬ M- page 83


https://www.facebook.com/groups/1639727976293954
Prometric Exam Collection 2019 01‫ودالشمال‬

.239
Patient arrived to ER from road accident, when the nurse assesse the
patient should a wear to: Immobilize the head

.241
Pregnant woman comes to gynaecological with vaginal bleeding dark
blood and abdominal cramp, when the doctor assessment found fatal
death, what type of abortion?

a. Complete abortion
b. In complete abortion
c. Threating abortion
d. Missing abortion
Missed abortion is early fetal intrauterine death without expulsion of the
products of conception

.240
The nurse staff informs the head nurse that one nurse haven’t work
properly in the ward the head nurse do?
a. Observation
b. Investigate problem
.242
After appendectomy the patient have distension and no bowel sound,
what the nurse do?

a. Check drain
b. Ambulate the patient
.243
Patient have blood transfusion, the nurse take it from blood bank and
left it in the ward more than four hours, the blood become:

a. Hypokalaemia
b. Hyperkalaemia
c. Hypernatremia
d. Hyponatremia
Hyperkalaemia is a common complication in transfusion of stored
blood. The supernatant of stored RBCs usually contains more than 60

‫ مناقشة امتحانات البرومتريك للتمريض‬.. ‫قروب طور ذاتك‬ M- page 84


https://www.facebook.com/groups/1639727976293954
Prometric Exam Collection 2019 01‫ودالشمال‬

mEq/L of potassium. Potassium in stored blood increases due to


decrease in ATP production and leakage of potassium into the
supernatant.
.244
Newborn-Reflexes

What reflexes should be present in a newborn?

Reflexes are involuntary movements or actions. Some movements are


spontaneous, occurring as part of the baby's usual activity. Others are
responses to certain actions. Reflexes help identify normal brain and nerve
activity. Some reflexes occur only in specific periods of development. The
following are some of the normal reflexes seen in newborn babies:

Root reflex. This reflex begins when the corner of the baby's mouth is
stroked or touched. The baby will turn his or her head and open his or her
mouth to follow and "root" in the direction of the stroking. This helps the
baby find the breast or bottle to begin feeding.

Suck reflex. Rooting helps the baby become ready to suck. When the
roof of the baby's mouth is touched, the baby will begin to suck. This
reflex does not begin until about the 32nd week of pregnancy and is not
fully developed until about 36 weeks. Premature babies may have a weak
or immature sucking ability because of this. Babies also have a hand-to-
mouth reflex that goes with rooting and sucking and may suck on fingers
or hands.

Moro reflex. The Moro reflex is often called a startle reflex because it
usually occurs when a baby is startled by a loud sound or movement. In
response to the sound, the baby throws back his or her head, extends out
the arms and legs, cries, then pulls the arms and legs back in. A baby's
own cry can startle him or her and trigger this reflex. This reflex lasts
about 5 to 6 months.

Tonic neck reflex. When a baby's head is turned to one side, the arm on
that side stretches out and the opposite arm bends up at the elbow. This is

‫ مناقشة امتحانات البرومتريك للتمريض‬.. ‫قروب طور ذاتك‬ M- page 85


https://www.facebook.com/groups/1639727976293954
Prometric Exam Collection 2019 01‫ودالشمال‬

often called the "fencing" position. The tonic neck reflex lasts about 6 to
7 months.

Grasp reflex. Stroking the palm of a baby's hand causes the baby to close
his or her fingers in a grasp. The grasp reflex lasts until about 5 to 6
months of age.

Babinski reflex. When the sole of the foot is firmly stroked, the big toe
bends back toward the top of the foot and the other toes fan out. This is a
normal reflex up to about 2 years of age.

Step reflex. This reflex is also called the walking or dance reflex because
a baby appears to take steps or dance when held upright with his or her
feet touching a solid surface.

‫ مناقشة امتحانات البرومتريك للتمريض‬.. ‫قروب طور ذاتك‬ M- page 86


https://www.facebook.com/groups/1639727976293954
Prometric Exam Collection 2019 01‫ودالشمال‬

.245
The mother talk the nurse she want to do bottle feeding for her baby
because she is working, what the nurse advise him?

The important of bottle clean before use

.246
The central line catheter position >>>>> trendelenburg’s position

.247
Study for organism interact with their environment

a. Ecology
b. pathology
c. microbiology
Ecology is the study of how organisms interact with their environment,
including both abiotic (non-living) and biotic (living) aspects of the
environment. It is a very broad definition and the science of ecology tends to
overlap other biological sciences

.248
Patient in CCU unit, the nurse note that absent of P wave and t wave
depression in patient ECG, the nurse understand that patient have
ventricular fibrillation and need to:

Defibrillation

‫ مناقشة امتحانات البرومتريك للتمريض‬.. ‫قروب طور ذاتك‬ M- page 87


https://www.facebook.com/groups/1639727976293954
Prometric Exam Collection 2019 01‫ودالشمال‬

.249
After operation the patient inform the head nurse she lost her jewel,
after minutes the nurse found it in house keeper bag what the head
nurse do?

a. Group counselling
b. Ask housekeeper
c. Inform security
.251
16 year old female known case of bipolar disorder, she is aggressive and
agitated. What the nurse do?

Put him in environment quite

.250
The Foley Family is caring for their youngest child, Justin, who is
suffering from tetralogy of Fallot. Which of the following positions is
used for congenital heart condition?

a. Semi fowler's position


b. knee chest position
c. semi position
There are simple procedures such as squatting and the knee chest position
which increase systemic vascular resistance and decrease right-to-
left shunting of deoxygenated blood into the systemic circulation.

‫ مناقشة امتحانات البرومتريك للتمريض‬.. ‫قروب طور ذاتك‬ M- page 88


https://www.facebook.com/groups/1639727976293954
Prometric Exam Collection 2019 01‫ودالشمال‬

Tetralogy of Fallot (TOF) is a type of heart defect present at


birth. Symptoms at birth may vary from none to severe. Later there is
typically an episode of bluish color to the skin. When affected babies cry or
have a bowel movement, they may develop a "tet spell" where they turn very
blue, have difficulty breathing, become limp, and occasionally lose
consciousness. Other symptoms may include a heart murmur, finger
clubbing, and easy tiring upon breastfeeding.

Classically there are four defects:


 pulmonary stenosis, narrowing of the exit from the right ventricle

 a ventricular Septal defect, a hole between the two ventricles

 right ventricular hypertrophy, thickening of the right ventricular


muscle

 an overriding aorta, which allows blood from both ventricles to enter


the aorta

.252
Patient has William disease with have abdomen tenderness & mass the
nurse should Avoid:

a. Sleeping patient in lateral position


b. Palpate abdomen
c. Check mouth of baby every 30 min
.253
Pregnancy in 20 week has abdomen cramping and vaginal bleeding and
amino fluid leaking, what the type of abortion?
a. Missed
b. Incomplete
c. Threating
.254
Patient post c/s delivery diagnosis DM has baby well good sucking, what
the first action for nurse to the baby?

a. Monitoring hypoglycemia
b. Maintaining oral feeding

‫ مناقشة امتحانات البرومتريك للتمريض‬.. ‫قروب طور ذاتك‬ M- page 89


https://www.facebook.com/groups/1639727976293954
Prometric Exam Collection 2019 01‫ودالشمال‬

c. Ringer lactate
.255
28 years old patient hyperkalaemia, respiratory rate 11, pulse rate 130
and have severe oliguria, this patient should treat by:
a. Sodium resonium
b. Narcan
Resonium A (sodium polystyrene sulfonate) is a type of medicine used to
help remove excessive amounts of potassium from the blood.

.256
17 year old mother after normal delivery have anemia and bleeding
which of the following education the nurse do?
a. Mass media
b. Group education
c. Individual education
.257
Patient have severity pain take medication to relieved pain after that
nurse come to assess patient found patient deep sleeping and no
movements what the nurse first do?

a. Awake patient
b. Conscious level
c. Start CPR
.258
The nurse come to do caring to many patients in the unit, any patient of
the following have more priority?

a. Patient has dressing


b. Appendectomy patient received immediately

.259
Patient comes to ER with lower guardant pain, nausea and vomiting
and diagnosis with appendicitis, what the priority nurse diagnosis

a. Acute pain
b. Vomiting

‫ مناقشة امتحانات البرومتريك للتمريض‬.. ‫قروب طور ذاتك‬ M- page 90


https://www.facebook.com/groups/1639727976293954
Prometric Exam Collection 2019 01‫ودالشمال‬

.261
Thoracentesis
Thoracentesis is a procedure in which a needle is inserted into the pleural
space between the lungs and the chest wall. This procedure is done to
remove excess fluid, known as a pleural effusion, from the pleural space to
help you breathe easier

Post Thoracentesis position:

Patients’ positions described in the text:

A. Supine with head and chest elevated at 30-45°;


B. Lateral recumbent with head and chest elevated at 30-45°.
Black arrows indicate the site of pleural procedures.

‫ مناقشة امتحانات البرومتريك للتمريض‬.. ‫قروب طور ذاتك‬ M- page 91


https://www.facebook.com/groups/1639727976293954
Prometric Exam Collection 2019 01‫ودالشمال‬

.260
The nurse was evaluating nursing care plan for a 50 year-old female
patient who has been scheduled for lithotripsy due to urinary calculus.
Which of the following is the priority evaluation?
a. edema
b. acute pain
c. knowledge deficits
d. urinary tract infection
Nursing Interventions Rational

Determine and note location, duration, Aids to evaluate site of obstruction and
intensity (0–10 scale), and radiation. progress of calculi movement. Flank pain
Document nonverbal signs such as suggests that stones are in the kidney area,
elevated BP and pulse, restlessness, upper ureter. Flank pain radiates to back,
moaning, thrashing about. abdomen, groin, genitalia because of
proximity of nerve plexus and bloodvessels
supplying other areas. Sudden, severe pain
may precipitate apprehension, restlessness
and severe anxiety

.262
42 years old man prepared to cholecystectomy and he sign consent, this
action of informed consent means:

a. Autonomy
b. Confidently
c. Justice

.263
Nursing Supervisor assessment one of staff nurse for three month in
outpatient, this consider?

‫ مناقشة امتحانات البرومتريك للتمريض‬.. ‫قروب طور ذاتك‬ M- page 92


https://www.facebook.com/groups/1639727976293954
Prometric Exam Collection 2019 01‫ودالشمال‬

a. Evaluation
b. Feedback
.264
Education patient post vaginal delivery for breast feeding engorgement?

a. Mother breast feeding 2-3hour at day and night


b. Mother breast feeding via bottle at night
c. Mother breast feeding 2-3 hour at day just never at night breast
feeding
d. Every 1.5-2 hours during the day and at least every 2-3 hours at
night until engorgement is gone
Breastfeed at least every 1.5-2 hours during the day and at least every 2-
3 hours at night until engorgement is gone. Use breast massage or
compression during feedings to more fully drain breasts. Let warm water run
over breasts in the shower. Leaking relieves pressure.

.265
Patient diagnosis Nephrotic syndrome what is positive sign detect
patient treated?

a. Low protein urea


b. Haematuria
.266
5 years old boy comes to paediatrician with his parent, have injury and
bruises, the nurse to know what happen to child, what she do?

a. Ask the parent


b. Encourage the child to talk
.267
Angina pectoris treatment >>>>>Rest and decrease cholesterol in diet

.268
10 years old boy diagnosis appendicitis has serve pain, the tool of pain
scale to be use is:

a. Face pain rating


b. Flacc rating scale

‫ مناقشة امتحانات البرومتريك للتمريض‬.. ‫قروب طور ذاتك‬ M- page 93


https://www.facebook.com/groups/1639727976293954
Prometric Exam Collection 2019 01‫ودالشمال‬

c. Nonverbal rating scale

‫ مناقشة امتحانات البرومتريك للتمريض‬.. ‫قروب طور ذاتك‬ M- page 94


https://www.facebook.com/groups/1639727976293954
Prometric Exam Collection 2019 01‫ودالشمال‬

.269
What of the following correct statement in about clomid drug?

a. Clomid dose 50 MG given orally for ovulation


b. Clomid cause multi pregnant
.271
30 years old women absent her menstruation for 5 months and her
menstrual cycle come every 28 days, she controlled her diet and do
heavy exercise this women condition?

a. Pregnancy
b. Primary amenorrhea
c. Secondary amenorrhea
Types of absent menstruation
 The two types of amenorrhea:
 Primary amenorrhea: is when a teenage girl has reached or
passed the age of 16 and still hasn’t had her first period. Most girls

‫ مناقشة امتحانات البرومتريك للتمريض‬.. ‫قروب طور ذاتك‬ M- page 95


https://www.facebook.com/groups/1639727976293954
Prometric Exam Collection 2019 01‫ودالشمال‬

begin menstruating between ages 9 and 18, but 12 is the average


age.
 Secondary amenorrhea: is when a woman has stopped
menstruating for at least three months. This is the more common
form of amenorrhea.
In most cases, both types can be treated effectively.

.270
A physician orders an intravenous fluid of D5NS at 100cc/hr. This is an
example of which of the solution?

a. Hypotonic
b. Isotonic
c. Hypertonic
d. Hyper alimentation
.272
A physician orders Lactated Ringer Solution to infuse at 125 cc/hour.
This is an example of which type of solution?

a. Hypotonic
b. Isotonic
c. Hypertonic
d. Hyper alimentation
.273
The primary difference between practical nursing licensure and a
nursing certification in an area of practice is that nursing licensure is:

a. Insures competency and a nursing certification validates years of


experience.

b. Mandated by the American Nurses Association and a nursing


certification are not.

c. Is legally mandated by the states and a nursing certification is not.

d. Renewed every two years and a nursing certification is renewed every


five years.

‫ مناقشة امتحانات البرومتريك للتمريض‬.. ‫قروب طور ذاتك‬ M- page 96


https://www.facebook.com/groups/1639727976293954
Prometric Exam Collection 2019 01‫ودالشمال‬

.274
A community nurse started a campaign to stop a factory from disposing
chemical waste in the river that is running directly to poor population
housing area. What does the community nurses action indicate?
a. Societal justice
b. Non-maleficence
c. Environmental justice
.275
A newly graduated nurse is inserting an intravenous cannula into the
mid-cephalic vein of a patient who is being admitted to the unit. As she
withdraws the needle, a nurse calls out for help from another patient’s
room. The new nurse rushes to help. She secured the intravenous
catheter and threw the needle into the waste basket instead of the
sharp’s container. When asked who had thrown the needle into the bin,
the new nurse admits that she had made that mistake. Which
professional act best describes the newly graduated nurse’s response?
a. Responsibility
b. Accountability
c. Assertiveness
d. Leadership
.276
A five year-old child was brought to the Emergency Room with a
fractured right forearm. He had several bruises on his body but showed
no signs of pain while palpating them. He seemed scared and did not
answer any questions asked. Why should the nurse discuss this case
with the nurse manager?
a. Continuity of care
b. Rule out child abuse
c. Psychological support
d. Fracture management
.277
A 78 year-old man with a diagnosis of Alzheimer’s disease had been
wandering outside the home and made an attempt to escape the
compound of the long-term care facility where he lived. When the
nurses approached him during such an escape he becomes aggressive
and combative. Once back inside, he would not calm down and after
several attempts reason with him, he is placed in restraints on his bed. A

‫ مناقشة امتحانات البرومتريك للتمريض‬.. ‫قروب طور ذاتك‬ M- page 97


https://www.facebook.com/groups/1639727976293954
Prometric Exam Collection 2019 01‫ودالشمال‬

nurse is assigned to provide one-to-one care in the immediate period


afterwards. What is the next most important step?
a. Administer sedative as needed.
b. Monitor fluid and hydration status.
c. Check policies restraint duration
d. Ensure correct placement of restraints
.278
34 years-old women patient newly diagnosed with breast cancer asked
the nurse to don't share this data about her diagnosis to anyone except
her husband the nurse follows the patient is instruction according to the
previous situation the nurse applied which one of the following ethical
consideration
a. Autonomy
b. Confidentiality
c. Fairness
d. Beneficence
.279

Wilms’ tumor is a form of:

a. Renal cancer.
b. Liver cancer.
c. Basal cell carcinoma.
d. Brain cancer.
.281
An 18-year-old girl was under weight and length 163 cm, her body
weight was 45 Kg and her admitted in the Female Medical Ward for
severe observed that on the second day she inducing Dinner. On asking,
she replied that inducing helps clear the stomach bacteria. Which aspect
of the nursing care area is altered?
a. Self-concept
b. Health perception
c. Value-belief system
d. Nutrition management

‫ مناقشة امتحانات البرومتريك للتمريض‬.. ‫قروب طور ذاتك‬ M- page 98


https://www.facebook.com/groups/1639727976293954
Prometric Exam Collection 2019 01‫ودالشمال‬

.280
A 58-year-old man admitted to CCU unit have rapid and irregular
heartbeat, Swelling (edema) in legs, ankles and feet, Fatigue and
weakness. After assessment the test result normal value, Magnesium 2.8
0.7-1.2mmoI/L. Which an ECG changes is the nurse expected to note?
a. Multiple P waves
b. Prominent U waves
c. Prolonged QRS
d. Depressed ST segment
.282
You have been assigned to care for a neonate who has been diagnosed
with the Tetralogy of Fallot. The mother asks you what the Tetralogy of
Fallot is. How should you respond to this mother?

a. ―The Tetralogy of Fallot is a congenital gastrointestinal disorder‖

b. ―The Tetralogy of Fallot is a congenital cardiac disorder”

c. ―The Tetralogy of Fallot will affect the baby’s reflexes‖

d. ―The Tetralogy of Fallot will affect the baby’s ability to breastfeed‖

.283
A 28 year old man diagnosed HIV planned to discharge from Isolation
unit after he treated, which of the following is important nursing
diagnosis for the patient?
a. Risk of infection due to altered immune
b. Fluid volume deficit due to frequent Diarrhea
c. Anxiety due to disease, fear and social
d. Weight loss due to higher metabolism rate
.284
A young girl was scheduled for surgery and prepared, and all pre-
operative preparation should be done, but her nail polish and trims her
long nails and she refuse to clean it, the nurse response is:
a. Respect the patient's right to refuse
b. Explain why nails need to be cleaned
c. Record and inform physician
d. Remove as per protocol

‫ مناقشة امتحانات البرومتريك للتمريض‬.. ‫قروب طور ذاتك‬ M- page 99


https://www.facebook.com/groups/1639727976293954
Prometric Exam Collection 2019 01‫ودالشمال‬

.285
A53-year-old patient who was referred to the primary healthcare center
for hyperglycaemia him and finds out that he has been recently says
that he does not understand why he needs
Test Result Glucose, fasting 12.2 Normal Values 3.5-6.5 mm
Which of the following learning needs is indicated?
a. Insulin Alternatives
b. Complications of diabetes
c. Disease process of diabetes
d. Lifestyle and dietary changes
.286
An old man diagnosed meningitis arrived to ER. Healthcare providers
plan to precautions for the patient, what is the best nursing practice for
this case?
a. Use N95 mask
b. Monitor the negative air pressure
c. Use respiratory protective equipment all the
d. Keep the patients with the same diagnosis
.287
A patient is admitted with a subarachnoid haemorrhage complaining of
severe headache, nuchal rigidity and projectile vomiting. Lumbar
puncture would be contraindicated in this patient in which of the
following circumstances?

a. Vomiting continues
b. Intracranial pressure is increased
c. The person needs mechanical ventilation
d. Blood is anticipated in the cerebrospinal fluids
.288

A 9-month-old child who has had four ear infections in the past 6
months is being discharged. Which statement by the parent indicates
the need for further discharge teaching?

a. I should never put my baby to bed with bottle

‫ مناقشة امتحانات البرومتريك للتمريض‬.. ‫قروب طور ذاتك‬ M- page 100


https://www.facebook.com/groups/1639727976293954
Prometric Exam Collection 2019 01‫ودالشمال‬

b. My child should not use a pacifier after age 6 months


c. My child should drink his bottle while lying flat in my lap
d. My child should not be around people who smoke

.289
A patient had a total abdominal hysterectomy 2days-ago and has not
been out of the bed yet. The patient is complaining left leg pain and
swelling. What should the nurse do FIRST?

a. Gently massage the patient’s leg


b. Assess the patient’s pain level
c. Assess the patient for Homan’s sign
d. Instruct the patient to reflex the left knee and hip

.291
While caring for a patient in the post-anesthesia care unit (PACU), a
nurse plans to Keep the patient warm. What is the MUST important
reason for this action?
a. To preserve nutritional stores
b. To prevent cutaneous vessel dilation
c. To decrease patient anxiety
d. To lower risk of infection resulting from chill
.290
The nurse is teaching the mother of a 3-months-old infant about bottle
feeding. Which statement indicates the mother understands of
appropriate procedure?
a. “I should hold my baby in as lightly reclined position, close to my
body”
b. ―It is OK to prop the bottle on a pillow‖. ―
c. It can feed my baby whole milk‖
d. ―I should warm the bottles in the microwave if they come out of the
Refrigerator‖.

‫ مناقشة امتحانات البرومتريك للتمريض‬.. ‫قروب طور ذاتك‬ M- page 101


https://www.facebook.com/groups/1639727976293954
Prometric Exam Collection 2019 01‫ودالشمال‬

.292
A patient had a vitrectomy and is about to be transported to the post
anesthesia care unit (PACU). The patient should be placed in which of
the following positions before transport to the PACU?

a. Semi-fowler
b. Prone
c. Dorsal
d. recumbent
.293
While caring for a patient in the post-anesthesia care unit (PACU) Who
has developed Hypovolemic shock, a nurse should position the patient:

a. Flat with legs elevated


b. In Trendelenburg’s position
c. With the head of the bed elevated 45 degrees
d. Completely flat.
.294
While caring for a patient in the post-anesthesia care unit (PACU), a
nurse observes the onset of rapid breathing cyanosis, and narrowing
blood pressure. The nurse should plan to:

a. Administer bolus glucose


b. Suction the airway
c. Turn the patient to the right side
d. Administer intra venous fluids
.295
A patient who had abdominal surgery is in the post anesthesia care unit
(PACU).Which of the following nursing diagnosis takes PRIORITY?

a. Disturbed sleep pattern


b. Acute pain
c. Risk for infection
d. Ineffective airway clearance

‫ مناقشة امتحانات البرومتريك للتمريض‬.. ‫قروب طور ذاتك‬ M- page 102


https://www.facebook.com/groups/1639727976293954
Prometric Exam Collection 2019 01‫ودالشمال‬

.296
A patient is diagnosed with pulmonary hypertension. Which of the
following nursing diagnoses should be the PRIORITY?

a. Impaired gas exchanged related to altered blood flow secondary to


pulmonary capillary constriction.
b. Fatigue related to hypoxia.
c. Anxiety related to illness and loss of control.
d. Activity intolerance related to imbalance between oxygen supply
and demand due to right and left ventricular failure
.297
A patient with a pulmonary embolus and a nursing diagnosis of
impaired gas exchange has an order to obtain arterial blood gases. The
FIRST intervention by the nurse is to:

a. Perform an Allens test


b. Explain the procedure
c. Gather the equipment
d. Document the procedure

.298
When conducting discharge teaching for the parent of a child newly
diagnosed with cystic fibrosis. Which of the following statement by the
parent indicates the need for further teaching?

a. Weekly weights help evaluate effectiveness of nutritional interventions


b. Weekly weights help the doctor know if may child is absorbing nutrients
c. Weekly weights reassure my child that recovery is progressing
d. Weekly weights help the doctor know if my child needs additional
enzymes
.299
A patient with poor wound healing and poor appetite has an order to
begin total parental nutrition (TPN). Waiting for the TPN solution to
arrive from the pharmacy, the nurse should obtain:

‫ مناقشة امتحانات البرومتريك للتمريض‬.. ‫قروب طور ذاتك‬ M- page 103


https://www.facebook.com/groups/1639727976293954
Prometric Exam Collection 2019 01‫ودالشمال‬

a. Pair of sterile gloves


b. An infusion pump
c. IV tubing with a micro-drip chamber
d. Povidine-iodine (Beta dine) swabs
.311
A patient schedule for a major surgery in one hour is very nervous and
upset. Which of the following order medications would the nurse
administer to relax this patient?
a. Meperidine Hydrochloride(Demerol)
b. Scopolamine (Transderm-Scop)
c. Pentobarbital sodium(Nembutal sodium)
d. Trazodone hydrochloride(Trazadone)
.310
The nurse administered a dose of morphine sulfate as prescribed to a
patient who is in the post anesthesia care unit (PACU). The patient
appears to be resting comfortably, the respiratory rate is 8 and the
O2saturation is 21 oxygen via cannula is 86%. The nurse should
IMMEDIATELY administer:
a. Flumazenil (Romazicon)
b. Medazolum (versed)
c. Naloxone (Narcan)
d. Ondansetron (Zofran)
.312
A surgeon instructs a nurse to serve as a witness to an elderly patient’s
informed consent for surgery. During the explanations to the patient, it
becomes clear that the patient is confused and does not understand the
procedure, but reluctantly sign the consent form. The nurse should:

a. Sign the form as a witness, making a nation that the patient did not
appear to understand
b. Not sign the form as a witness and notify the nurse supervisor
c. Not sign the form and answer the patient’s questions after the surgeon
leaves he room

‫ مناقشة امتحانات البرومتريك للتمريض‬.. ‫قروب طور ذاتك‬ M- page 104


https://www.facebook.com/groups/1639727976293954
Prometric Exam Collection 2019 01‫ودالشمال‬

d. Sign the form and tell surgeon that the patient doesn’t understand the
procedure.
.313
Shrinkage device is applied after surgery for amputation of the leg. The
goal of the shrinkage device is to from the residual limb into what
shape?

a. Cone
b. Oval
c. Mushroom
d. Cylinder with blunt end

.314
Following lumbar surgery a patient has a 4 millimeter (mm) surgical
incision. The incision is clean and the edges are well appropriate. This
type of tissue healing is classified as which of the following?
a. Primary intention
b. Secondary intention
c. Tertiary intention
d. Superficial epidermal
.315
A nurse plans to teach a group of 20to25-year-old women about oral
contraceptives. The nurse should instruct that oral contraceptives may:

a. Increase the risk of pelvic inflammatory disease


b. Cause acne to worsen
c. Decrease the risk of breast and cervical cancer
d. Decrease the risk of endometriosis
.316
A 16-years old patient present to the clinic requesting birth control.
With the diagnosis of health seeking behaviors, the BEST goals have the
patient:

‫ مناقشة امتحانات البرومتريك للتمريض‬.. ‫قروب طور ذاتك‬ M- page 105


https://www.facebook.com/groups/1639727976293954
Prometric Exam Collection 2019 01‫ودالشمال‬

a. Verbalizing understanding of safe sex practices and following safe sexual


practices in all encounters
b. Not engaging in sexual encounters until she is over18 years old and
maintaining a healthy life style
c. Recognizing the sign of pregnancy and the symptoms of sexually
transmitted diseases.
d. Understanding safe sexual practices and use a condom to prevent
pregnancy and sexually transmitted diseases
.317
A home health nurse visits a patient with diabetes and primary open-
angle glaucoma. The patient takes metformin (Glucophage) 500 mg
once a day for diabetes and timolol ophthalmic solution twice a day in
each eye for glaucoma. Which of the following evaluations indicates that
the patient is noncompliant with glaucoma management?
a. Patient has not been taking Glucophage
b. Patient has tearing of the eye
c. Patient has not refilled prescription for timolol in 3 months
d. Patient has yellow discharge from the
.318
To decrease the incidence of aspiration of gastric contents in a child
hospitalization with severe burns, the nurse should position the head:
a. Flat except during meals
b. Elevates 30-45 degrees during meals
c. Elevated 15-30 degrees for12-hours after meals
d. Elevated 45 degrees at all times
.319
A patient had a craniotomy with resection of a nonmalignant neoplasm
for the temporal lobe. The patient’s vital signs are within the base line
normal range. The nurse observes that the patient has developed
bilateral per orbital edema. Which of the following actions would be
appropriate for the nurse to take?
a. Apply cold compresses to the patient’s eyes
b. Apply warm compresses to the patient’s eyes
c. Elevate the head of the patient’s bed to 60 degrees

‫ مناقشة امتحانات البرومتريك للتمريض‬.. ‫قروب طور ذاتك‬ M- page 106


https://www.facebook.com/groups/1639727976293954
Prometric Exam Collection 2019 01‫ودالشمال‬

d. Elevate the head of the patient’s bed to 45 degrees


.301
A home health nurse is teaching a family member about the care of
patient’s peripherally inserted central catheter (PICC). Which of the
following statements would be appropriate for the nurse to make?

a. Place the used intravenous tubing in a leak proof container and then
place this sealed container inside a second leak proof container.”
b. You will need to put on a disposable face mask before you connect the
intravenous tubing to the port of the PICC.
c. The port of the PIC catheter will need to be cleansed with povidone-
iodine (BETADINE) after the infusion is completed.‖ ―
d. The empty medication container can be placed in the same container as
your Household refuses.‖
.300
A home health nurse has entered a home to complete an admission
assessment on a patient who has a methicillin-resistant Staphylococcus
aureus (MRSA) urinary tract infection. The patient will receive
intravenous anti-infective via a peripherally inserted central catheter
(PICC) for 3 weeks. Which of the following actions should the nurse
take FIRST?

a. Shake the patient’s hand


b. Place the nursing supply bagon a clean, dry surface.
c. Obtain the patient’s written consent for home health care
d. Perform hand hygiene per the agency protocol
.302
A patient admitted with a cerebrovascular accident (CVA), is unable to
chew or swallowed. The patient is a risk for aspiration. The nurse would
anticipate receiving which of the following orders for this patient?

a. Give no food by mouth and start intravenous hydration


b. Start a pureed diet with thickened liquids
c. Refer the patient to a psychiatrist for depression related to the CVA

‫ مناقشة امتحانات البرومتريك للتمريض‬.. ‫قروب طور ذاتك‬ M- page 107


https://www.facebook.com/groups/1639727976293954
Prometric Exam Collection 2019 01‫ودالشمال‬

d. Refer the patient to physical therapy for muscle strengthening


.303
A home care nurse makes a follow-up visit to a patient who recently
suffered a cerebrovascular accident. The patient is mobile and able to
perform activities of daily living. However, the patient has not sleeping
and has lost weight due to lack of appetite. The patient also feels
overwhelmed with sadness. Which of the following is the most
appropriate evaluation?

a. Patient’s progress is as expected and no further intervention is


necessary
b. Patient needs referral to a nutritionist
c. Patient needs intervention for depression
d. Patient needs sleeping medication
.304
A nurse is assigned to do a home visit for an 81-year-old patient. The
patient lives at home with an adult caretaker and is completely bed-
bound following a Cerebrovascular accident (CVA) 2 weeks ago. In
planning caregiver education, The nurse should be prepared to instruct
the caretaker in:

a.How to select a nursing home for the patient.


b.Performing passive range of motion exercises.
c.The importance of avoiding viscous drinks.
d.Forming a local chapter of a care giver support group
.305
A home health nurse is visiting a patient who recently suffered a
Cerebrovascular accident (CVA). The nurse would MOST likely
implement which of the following interventions to prevent muscle and
ligament deformities?

a. Daily moist heat and isometric exercises


b. Daily balance training and routine medications for pain

‫ مناقشة امتحانات البرومتريك للتمريض‬.. ‫قروب طور ذاتك‬ M- page 108


https://www.facebook.com/groups/1639727976293954
Prometric Exam Collection 2019 01‫ودالشمال‬

c. Instruct patient to use non-affected side to perform activities of daily


living
d. Daily range of motion exercises.
.306
A home health nurse is visiting a patient following a cerebrovascular
accident (CVA). The patient is having trouble sleeping and is feeling
sad. The patient’s spouse tells the nurse that the patient is not eating
much and often cries when nooneis watching. Which of the following
would be the nurse’s MOST likely intervention?
a. Assess for changes in cognitive abilities
b. Complete a depression index
c. Strengthen family coping methods
d. Screen for pain

.307
A community health care nurse visits a patient who had
cerebrovascular accident. The patient is at risk for deficient volume due
to voluntary reduction intake fluid intake to avoid the use of the
bathroom. The nurse educates the patient on the importance of
drinking fluids and maintaining hydration. Which of the following
indicates the efficacy of the nursing intervention?
a. Amber color urine
b. Respiration of 35
c. Tachycardia
d. Moist mucous membrane

.308
When teaching a community class on cerebrovascular accidents
(stroke), which of the following should participants of the class know at
the completion of the class?
a. Muscle and ligament damage is not reversible
b. Expressive aphasia is resolved by voice rest
c. There is a risk for mood disorders such as depression
d. Liquids should be consumed at the same times as solids food

‫ مناقشة امتحانات البرومتريك للتمريض‬.. ‫قروب طور ذاتك‬ M- page 109


https://www.facebook.com/groups/1639727976293954
Prometric Exam Collection 2019 01‫ودالشمال‬

.309
During surgery requiring general anesthesia, the patient heart’s stops
and a carotid pulse is not palpated. How many compressions per minute
should be administered?

a. 50
b. 60
c. 80
d. 100
.321
While caring for a child with aventriculoperitoneal shunt revision, the
nurse find the patient lying with the head and feet flexed back. The
nurse should call for help and prepare for a(n):

a. Spinal tap
b. Shunt culture
c. Electrocardiogram
d. Ventricular tap

.320
The nurse is caring for a patient who had major abdominal surgery
under general anesthetic 4 hours ago. An appropriate goal for the
patient includes:

a. Having minimal fine crackles in the base of the lungs Using the
incentive spirometry every 4 hours
b. Expectorating minimal amount of secretions
c. Performing Coughing
d. Exercises every hour while awake
.322
A nurse visits a patient at home who does not understand how to take a
newly prescribed medication. The prescription reads: 5 ml PO TID p.c.
meals. The nurse explains to the patient that the correct way to take the
medication is:

‫ مناقشة امتحانات البرومتريك للتمريض‬.. ‫قروب طور ذاتك‬ M- page 110


https://www.facebook.com/groups/1639727976293954
Prometric Exam Collection 2019 01‫ودالشمال‬

a. 1 teaspoon by mouth, 3times a day, before meals


b. 1 teaspoon by mouth, 3times a day, after meals
c. 1 tablespoon by mouth, 3times a day, before meals
d. 1 tablespoon by mouth, 3times a day, after meals
.323
A 12-year-old child who has been diagnosed with insulin dependent
mellitus (IDDM) since age3.Comes to the clinic for a routine visit. The
patient has begun to self-manage care with parental supervision. The
patient injects 28 units of NPH insulin every morning and 8units at
bedtime. The patient checks blood sugar 4 times every day. The
patient’s weight is stable and diet is unchanged. However, the patient
reports several hypoglycemic reactions every week. The nurse knows
the MOST likely cause is that:
a. The patient is not eating the adequate number of calories reported
b. The dosages of insulin may need to be decreased as the patient
continues to grow
c. There may be changes in exercise or stress levels or the beginning
of a growth Spurt
d. The patient may not be competent in techniques of drawing up and
injecting insulin
.324
When giving post-operative discharge instructs a patient who had
abdominal surgery, all of the following regarding wound healing are
true EXCEPT:

a. Bathing to soak abdomen is preferred


b. Avoid tight belts and cloths with seams that may rub the wound
c. Pain medication may affect ability to drive
d. Irregular bowel habits can be expected
.325
The nurse is evaluating the patient with end stage chronic obstructive
pulmonary disease (COPD).The patient has not achieved any of the
goals in the plan of care. The spouse reports concerns about the

‫ مناقشة امتحانات البرومتريك للتمريض‬.. ‫قروب طور ذاتك‬ M- page 111


https://www.facebook.com/groups/1639727976293954
Prometric Exam Collection 2019 01‫ودالشمال‬

patient’s mood and increased dependency .What action should the


nurse take FIRST?
a. Continue the care plan for 1more month
b. Refer the patient to psychiatric services
c. Collaborate with the patient and spouse to revise the care plan
d. Revise the care plan based on the spouse’s input
.326
A nurse has just started total parenteral nutrition (TPN) as prescribed
for a patient with severe dysphagia low prealbumin levels. In one to two
hours, the nurse should anticipate assessing the patient’s:

a. Blood glucose level


b. Weight
c. Liver
d. Spo 2
.327
A nurse is caring for a patient receiving total parenteral nutrition
(TPN). The patient reports the sudden onset of feeling short of breath
and anxious. The nurse hears crackles in bilateral lower lobes of the
lungs and the patient’s O2 saturation is 90%on room air. The nurse
must IMMEDIATELY:
a. Turn off the TPN
b. Notify the physician
c. Asses the patient’s capillary blood glucose level
d. Attempt to suction the patient’s airway
.328
A home care patient with chronic obstructive pulmonary disease
(COPD) reports an upset stomach. The patient is taking
theophylline(Theo-Dur) and triamcinolone acetonide (Azmacort) The
nurse should instruct the patient to take:
a. Theo-dur an empty stomach
b. Theo-dur and Azmacort at the same time
c. Theo-dur and azmacort12 hours apart
d. Theo-dur milk or crackers

‫ مناقشة امتحانات البرومتريك للتمريض‬.. ‫قروب طور ذاتك‬ M- page 112


https://www.facebook.com/groups/1639727976293954
Prometric Exam Collection 2019 01‫ودالشمال‬

.329
While caring for a patient with an ileostomy, the nurse would expect the
ostomy to be located In Which Quadrant of the abdomen?
a. Right lower
b. Left lower
c. Left upper
d. Right upper
.331
A nurse is caring for a child with a diagnosis of cystic fibrosis and
pneumonia. The plan of care includes nebulizer treatment and chest
physiotherapy. The nurse should perform chest physiotherapy:
a. Continuously during the nebulizer treatment
b. Prior to the nebulizer treatment
c. After the nebulizer treatment
d. Intermittently during the nebulizer treatment
.330
A nurse is caring for an 8-year-old male with cystic fibrosis. Based on
the nurse’s understanding of the disease. What nursing intervention
should the nurse expect to perform?
a. Restrict sodium and fluid intake
b. Give antidiarrheal medications
c. Discourage coughing after postural drainage
d. Administer pancreatic enzymes with each meal
.332
The nurse is caring for a 4-year-old patient with a diagnosis of cystic
fibrosis and pneumonia. The child is feeling better on the 3rd day of the
hospitalization and “wants to play.” What would be the best choice of
entertainment?
a. Blowing bubbles
b. Looking at picture books
c. Watching videos
d. Riding in a wagon

‫ مناقشة امتحانات البرومتريك للتمريض‬.. ‫قروب طور ذاتك‬ M- page 113


https://www.facebook.com/groups/1639727976293954
Prometric Exam Collection 2019 01‫ودالشمال‬

.333
The nurse is planning care for several children who were admitted
during the shift. Daily weights should be the plan of care for the child
who is receiving:
a. Total parenteral nutrition (TPN)
b. Supplement oxygen
c. Intravenous anti-ineffective
d. Chest physiotherapy
.334
The nurse is visiting the asthmatic patient at home to reinforce the
importance of eliminating environmental allergens and to assess the
patient’s response to the environmental changes. This type of
implementation is called:
a. Supervision and coordination
b. Discharge planning
c. Monitoring and surveillance
.335
An asthmatic patient presents with wheezing and coughing. Oxygen
saturation is 88% on room air. Which of the following nursing diagnosis
would take priority?
a. Imbalanced nutrition related to decreased food intake
b. Activity intolerance related to inefficient breathing
c. Anxiety-related dyspnea and concern of illness
d. Ineffective gas exchange related to broncho spasm
.336
A Child is diagnosed with asthma exacerbation. Which of the following
nursing diagnoses should be the FIRST priority?
a. In effective airway clearance related to broncho spasm and
mucosal edema
b. Fatigue related to hypoxia.
c. Anxiety related to illness and loss of control
d. Deficient knowledge related to potential side effect of the medication.

‫ مناقشة امتحانات البرومتريك للتمريض‬.. ‫قروب طور ذاتك‬ M- page 114


https://www.facebook.com/groups/1639727976293954
Prometric Exam Collection 2019 01‫ودالشمال‬

.337
A nurse administers an albuterol nebulizer on a child with asthma
exacerbation. Which of following indicates effectiveness of the
treatment?

a. Adventitious breath sound with cough


b. O2 saturation 94%
c. Nasal flaring
d. Respiration rate 28
.338
What would be the long-term goal for a child with asthma?
a. Quickly reverse airflow obstruction
b. Correct hypoxemia
c. Deliver humidified oxygen via nasal cannula
d. Develop a home and school management plan
.339
A child with asthma is experiencing thick respiratory secretions
resulting in increased work of breathing. The best nursing intervention
is to:
a. Encourage fluids
b. Eliminate dairy products
c. Decrease relative humidity of the room
d. Have the child lay on the left side

.341
A 3-years-old has returned to the clinic 4 days after being diagnosed
with gastroenteritis and dehydration. A parent reports that the
vomiting has stopped, and the child is tolerating liquids, rice, apple
sauce, and bananas. The diarrhea persists, but seems to be decreasing in
volume. When evaluating for signs of dehydration, the nurse will assess
the patient’s skin turgor by:
a. Grasping the skin over the abdomen with two fingers raising the
skin with two fingers

‫ مناقشة امتحانات البرومتريك للتمريض‬.. ‫قروب طور ذاتك‬ M- page 115


https://www.facebook.com/groups/1639727976293954
Prometric Exam Collection 2019 01‫ودالشمال‬

b. Grasping the skin over the forehead with two fingers and raising the
skin with two fingers two fingers.
c. Holding the patient’s mouth open and assessing the tongue for deep
creases or Furrows.
d. Drawing two tubes of blood and running blood urea nitrogen (BUN)
and creatinine (Cr).

.340
In evaluating the appropriateness of various exercises enjoyed by a
patient with osteoporosis, the nurse would recommend:
a. Walking
b. Bowling
c. Sit-ups
d. Golf

.342
A 34 years old women comes to gynaecological complain of her breast
swollen, redness and pain when breast feeding, what the nursing advise
to her?

Instruct him to continue breast feeding from the affected side & and
drink lots of water & ibuprofen analgesic

.343
A patient with chronic obstructive pulmonary disease (COPD)
experiencing frequent dyspnea which of the following exercise would
teach the patient how to BETTER control breathing?
a. Lower side rib
b. Segmental
c. Pursed-lip
d. Diaphragmatic

‫ مناقشة امتحانات البرومتريك للتمريض‬.. ‫قروب طور ذاتك‬ M- page 116


https://www.facebook.com/groups/1639727976293954
Prometric Exam Collection 2019 01‫ودالشمال‬

.344
A nurse makes a home visit to a patient recently diagnosed with chronic
obstructive pulmonary disease (COPD), which of the following should
the nurse teach the patient about managing COPD?
a. Recognizing signs of impending respiratory infection
b. Limiting fluids intake minimize bronchial secretions
c. Correct technique to auscultate the lung fields
d. Importance of starting antibiotic therapy
.345
All of the following are types of spina bifida EXCEPT:
a. Myelomeningocele
b. Hemophilia
c. Meningocele
d. Spina Bifida Occulta

Myelomeningocele, meningocele, and spina bifida occulta are all types of


spina bifida. Hemophilia is an inherited bleeding disorder in which the
blood does not clot properly.
.346

Red has just returned from the post anesthesia care unit (PACU) from a
hemorrhoidectomy. His postoperative orders include sitz baths every
morning. The nurse understands that sitz bath is use for:
a. Cause swelling
b. Relieve tension
c. Lower body temperature
d. Promote healing

‫ مناقشة امتحانات البرومتريك للتمريض‬.. ‫قروب طور ذاتك‬ M- page 117


https://www.facebook.com/groups/1639727976293954
Prometric Exam Collection 2019 01‫ودالشمال‬

.347

When assessing neonate’s hydration, the nurse should check for the
skins:
a. Color
b. Tone
c. Moisture
d. Elasticity
.348

ORDERED: deltasone 7.5 mg now. AVAILABLE: deltasone 2.5 mg


scored tablets .How many should you give?
a. 1 ½ tablets
b. 1 tablets
c. 2 ½ tablets
d. 3 tablets
.349

Health teaching to a diabetic patient will be on:


a. about complications
b. diet , insulin , exercise , feeding
c. diet , hypoglycemic , exercise , feeding
d. diet, oral hypoglycemic , weight loss , feeding
.351

You heard the nurse talk to the doctor and he said “the patient has
macro Hematuria”. What he means by hematuria?
a. The patient cannot urinate
b. Difficulty of urination
c. The urine seems bloody
d. Difficult of breathing

.350

During pregnancy iron supplement starts on:


a. Second trimester
b. third trimester
c. First trimester

‫ مناقشة امتحانات البرومتريك للتمريض‬.. ‫قروب طور ذاتك‬ M- page 118


https://www.facebook.com/groups/1639727976293954
‫‪Prometric Exam Collection 2019‬‬ ‫ودالشمال‪01‬‬

‫‪.352‬‬

‫‪While teaching diabetic patient to give himself insulin, you should stress‬‬
‫‪that injections should not be given in any one spot more often than‬‬
‫‪every:‬‬
‫‪a. month‬‬
‫‪b. one week‬‬
‫‪c. two weeks‬‬
‫‪d. 36 hours‬‬

‫قطرة املاء تثكب احلجر ليس بالعنف ولكن بتواصل السكوط‬


‫ال تيأس ‪ ..‬ألن الصعب سيمر وأوقات األمل البد أن يعكبوا أوقات فرح‬
‫وال حياة بدون عكبات!‬
‫مع متنياتى‬
‫بالتوفيل للجميع‬
‫ودالشنال‪ 2109‬م‬

‫صغِرياّ‪ ،‬وَاجِزِِهمَابِاإلِحِشَاِن‬
‫كمَا رَبََّيَانِي َ‬
‫رلِي وَِلوَالِدَيََّ‪ ،‬وَارِ َحمُِومَا َ‬
‫مَ اغِفِ ِ‬
‫الَّلَوُ َّ‬

‫ت عفوّا وغُفِرَاناّ‬
‫إِحِشَاناّ‪،‬وَبِالشََّيَِّئَا ِ‬
‫أسألكم الرمحة على والدى‬

‫قروب طور ذاتك ‪ ..‬مناقشة امتحانات البرومتريك للتمريض‬ ‫‪M- page 119‬‬
‫‪https://www.facebook.com/groups/1639727976293954‬‬

Вам также может понравиться